SlideShare a Scribd company logo
1 of 19
Download to read offline
Cracking IAS Study Circle, Chennai. PRELIMS 2012 SERIES
Current Affairs - Questions
_______________________________________________________
Address: - Y-block. 3020/1. 3rd
street. 13th
main road. Anna Nagar. Chennai-40
Landmark: Behind Natesan Institute of Co-operative Management on SHANTI COLONY ROAD.
Call: us: 09884554654. www.crackingIAS.com
1 Consider the following statements.
1. First Spices park in India was established @
Chhindwara, Madhya Pradesh under the
ASIDE scheme.
2. Under the concept of the Spice Park, Spices
Board will lease out the lands available in the
Spice Park to PSU’s for developing their own
processing plants for value addition.
(a) Only 1 is true
(b) Only 2 is true
(c) Both 1 and 2 are true
(d) Neither 1 nor 2 is true
2 Who heads the heads expert committee on
sugar sector?
(a) Kaushik Basu
(b) Ashok Gulati
(c) C. Rangarajan
(d) None of these
3 Minerals Resource Book (MRB) is prepared by
the
(a) Ministry of Earth Sciences.
(b) Geological Survey of India (GSI).
(c) Indian Bureau of Mines (IBM).
(d) Ministry of Trade and Commerce.
4 Which is the first state in India to deploy the
Special Tiger Protection Force (STPF)?.
(a) Karnataka
(b) Madhya Pradesh
(c) Tamilnadu
(d) West Bengal
5 India gifted a helicopter in November 2009 as
part of an economic package and installed a
coastal surveillance radar system in April 2011,
also recently in early 2012 announced a new
line of credit of US$250 million and a grant of
US$ 20 million to a country. Which is the
country cited under reference?.
(a) Maldives
(b) Mauritius
(c) Nigeria
(d) Zimbabwe
6 Consider the following statements.
1. Fertilizer is currently the main item of
export from Belarus to India.
2. Potash is the main import item of India from
Belarus.
(a) Only 1 is true
(b) Only 2 is true
(c) Both 1 and 2 are true
(d) Neither 1 nor 2 is true
7 Which of the following is/are true about
“Princess Elisabeth Antarctica” in news
recently?
(a) It is conceived as a zero-emission station,
entirely powered by renewable energy.
(b) It is an Australian scientific polar research
station.
(c) Neither (a) Nor (b)
(d) Both (a) & (b)
8 Concerns over the effects of Tributyltin (TBT)
are often in news. Which of the following is/are
true w.r.t. Tributyltin (TBT)?.
(a) It is an ingredient in certain antifouling paints
used on ships.
(b) It is present in Disposable diapers.
(c) It causes hormonal problems.
(d) All the above.
9 Who authored “Glimpses of Indian Women
Hockey”?
(a) Ajay Maken
(b) Leandro Negre
(c) V.N.Balan
(d) K. Arumugam
10 Bangladesh and India formally began signing
of strip-maps of 4,156kms common boundary
to settle the long-running issue between the two
close door neighbours. Which of the following
is/are not the feature of “Strip Map”?
(a) It is an un-scaled drawing of a route, includes
critical points along the border. (b) It usually
incorporates distances, route-side features and
town facilities on a simple flip-over style map.
(c) Neither (a) Nor (b)
(d) Both (a) & (b)
11 Which of the following is/are false w.r.t.
Golden Quadrilateral?.
(a) Only National Highways are used in the
Golden Quadrilateral.
(b) The present status of Golden Quadrilateral
project is complete.
(c) Neither (a) Nor (b)
(d) Both (a) & (b)
12 Consider the following statements.
Cracking IAS Study Circle, Chennai. PRELIMS 2012 SERIES
Current Affairs - Questions
_______________________________________________________
Address: - Y-block. 3020/1. 3rd
street. 13th
main road. Anna Nagar. Chennai-40
Landmark: Behind Natesan Institute of Co-operative Management on SHANTI COLONY ROAD.
Call: us: 09884554654. www.crackingIAS.com
1. The Mid Day Meal Scheme is the largest
noon meal scheme in the world.
2. Mid-day meal Scheme tries to legitimize the
directive principles enshrined in our
constitution.
(a) Only 1 is true
(b) Only 2 is true
(c) Both 1 and 2 are true
(d) Neither 1 nor 2 is true
13 Mt Bevan iron ore Project is based at
(a) Canada
(b) USA
(c) Australia
(d) Germany
14 Which of the following Asian country has
recently set up a laboratory to conduct
research on medicines for astronauts during
space missions?.
(a) India
(b) Pakistan
(c) Indonesia
(d) China
15 Which of the following is/are not the feature of
The National Common Mobility Card (NCMC)
named 'More', which was unveiled by urban
development ministry?.
(a) One can also pay parking and toll fees using
this card.
(b) The card, once it becomes operational, can be
used across different modes of transport like
buses, Metro, Bus Rapid Transport System etc.
(c) Neither (a) Nor (b)
(d) Both (a) & (b)
16 Which of the following is/are false?.
(a) India currently depends on images from a
Canadian satellite as domestic remote sensing
spacecraft cannot take pictures of the ground
during cloud cover.
(b) RISAT-1 would be particularly useful in
Kharif season when cloud-covered atmosphere is
frequent.
(c) Neither (a) Nor (b)
(d) Both (a) & (b)
17 Which of the following is/are false w.r.t.
Review of the Economy 2011-12 presented by
Dr. C. Rangarajan, Chairman, Economic
Advisory Council to the Prime Minister?.
(a) Farm sector output growth in 2011/12 has
been strong.
(b) While the electricity sector has performed
well, manufacturing and construction have
disappointed.
(c) Neither (a) Nor (b)
(d) Both (a) & (b)
18 Where is “The City of Puerto Princesa” which
is among 'New 7 Wonders of Nature'?
(a) Denmark
(b)Venezuela
(c) Philippines
(d) Tanzania
19 Consider the following statements w.r.t.
National Girl Child Day.
1. National Girl Child Day is celebrated on 24
January every year since 2010.
2. 24 January was chosen because on this date
in 2010 that UN Women Forum was formed.
(a) Only 1 is false
(b) Only 2 is false
(c) Both 1 and 2 are false
(d) Neither 1 nor 2 is false
20 Indian Space Research Organisation has signed
a deal, believed to be valued at around Rs 100
crore, to launch an 800 kg satellite of
ASTRIUM. ASTRIUM is a/an
(a) Aerospace subsidiary of the European
Aeronautic Defence and Space Company (EADS).
(b) Wing of European Space Agency.
(c) Aerospace subsidiary of the Israeli Space
Agency
(d) Wing of Russian Federal Space Agency
21 Asia Cup Cricket Trophy 2012 was won by
(a) India
(b) Srilanka
(c) Pakistan
(d) Bangladesh
22 What are Pygmy marmosets?
(a) They are the world's smallest monkey.
(b) They are the closest relatives of bird in the
evolutionary tree.
(c) They were declared extinct recently by IUCN.
(d) None of these.
23 Where is Khadakpurna Irrigation Project?
(a) Chattisgarh
(b) Madhya Pradesh
Cracking IAS Study Circle, Chennai. PRELIMS 2012 SERIES
Current Affairs - Questions
_______________________________________________________
Address: - Y-block. 3020/1. 3rd
street. 13th
main road. Anna Nagar. Chennai-40
Landmark: Behind Natesan Institute of Co-operative Management on SHANTI COLONY ROAD.
Call: us: 09884554654. www.crackingIAS.com
(c) Uttarkhand
(d) Maharashtra
24 A Titanium sponge plant, first of its kind in
India is @ Chavara near Kollam in Kerala.
Which of the following is/are true in this
context?
(a) It utilizes the technology developed by
Defence Metallurgical Research Laboratory
(DMRL), Hyderabad a laboratory under DRDO
(Defence Research and Development
Organization).
(b) Vikram Sarabhai Space Centre, Trivandrum,
which is a major user of titanium sponge, has
funded the plant.
(c) Both (a) & (b).
(d) Neither (a) nor (b).
25 India's first aerotropolis or airport city project
is at
(a) West Bengal
(b) Chennai
(c) Delhi
(d) Hyderabad
26 The first non-OECD, non-Council of Europe
country to become a party to Multilateral
Convention on Mutual Administrative
Assistance in Tax Matters is
(a) India
(b) Pakistan
(c) Bangladesh
(d) Srilanka
27 Which state legislature in USA has passed a bill
prohibiting companies doing business with
Iran's energy industry from entering into
contracts with the state (i.e)'Iran Divestment
Act' in January 2012?.
(a) New Jersey
(b) New York
(c) Virginia
(d) Florida
28 Who is Jordan Romero who was in news
recently?
(a) Mountain Climber
(b) Bio-Chemist
(c) Aerospace specialist
(d) Economist
29 Match the following
List 1 List 2
A. Satinder Singh 1. Malaysia
Bajaj
B. Sultan Abdul
Halim
2. Writer
C. Jamil Ahmed 3. Atomic Energy
4. Jordan
Codes
A B C
(a) 3 1 2
(b) 2 1 3
(c) 3 1 4
(d) 2 1 4
30 What is the objective of NASA’s “SPADE
SYSTEM”?
(a) To deorbit Space-Junk
(b) To research on Saturn’s Ring system.
(c) To research on Solar storms.
(d) To establish an House in Mars for Human
living purpose
31 Scientists have for the first time captured
images of auroras above the giant ice planet
Uranus, finding further evidence of just how
peculiar a world that distant planet is. Identify
the correct statement(s).
(a) Unlike auroras on Earth, which can turn the
sky greens and purples for hours, the newly
detected auroras on Uranus appeared to only last a
couple minutes.
(b) These were detected by means of observations
from the Hubble Space Telescope.
(c) Neither (a) Nor (b)
(d) Both (a) & (b)
32 A3P scheme is a part of
(a) Gramin Bhandaran Yojana(GBY)
(b) Rashtriya Krishi Vikas Yojana
(c) Macro Management of Agriculture Scheme
(d) National Food Security Mission
33 Which of the following countries has been
requesting for Ghasuapara and Dalu in
Meghalaya on Indo-Bangladesh border as on
exit/entry points for its trade with
Bangladesh?.
(a) Bhutan
(b) China
(c) Nepal
(d) Myanmar
34 The Indian Coast Guard Ship Rani Abbakka,
the first of a series of five Inshore Patrol
Cracking IAS Study Circle, Chennai. PRELIMS 2012 SERIES
Current Affairs - Questions
_______________________________________________________
Address: - Y-block. 3020/1. 3rd
street. 13th
main road. Anna Nagar. Chennai-40
Landmark: Behind Natesan Institute of Co-operative Management on SHANTI COLONY ROAD.
Call: us: 09884554654. www.crackingIAS.com
Vessel (IPV) built at Hindustan Shipyard
Limited (HSL), was commissioned at
Visakhapatnam. Which of the following is/are
true about Rani Abbakka?.
(a) She fought the French.
(b) She fought the Dutch.
(c) She fought the British.
(d) She fought the Mughals.
35 Consider the following statements.
1. Jawai Dam is known for Crocodile
Sanctuary.
2. Jawai river is a tributary of Luni river.
(a) Only 1 is true
(b) Only 2 is true
(c) Both 1 and 2 are true
(d) Neither 1 nor 2 is true
36 Consider the following statements.
1. ‘Punjab Mail’ is the first train to achieve
distinction of entering its 100th
year on Indian
Railways broad gauge network.
2. “Punjab Mail” Train was the theme of
Indian Railways Republic Day Tableau in
2012.
(a) Only 1 is false
(b) Only 2 is false
(c) Both 1 and 2 are false
(d) Neither 1 nor 2 is false
37 Who is Ali Ukla Ursan?
(a) A writer
(b) An aerospace specialist
(c) A nuclear scientist
(d) An aide of Osama Bin Laden
38 When was the constitution amended to lower
the voting age from 21 years to 18 years?.
(a) 1988
(b) 1989
(c) 1987
(d) 1990
39 What is “Bheem” which was in news recently?.
(a) A newly designed diesel locomotive. (b) A
newly designed fuel efficient car.
(c) A newly formulated energy enhancer for
athletes by DRDO.
(d) A newly designed Battle Tank made by HVF
Avadi.
40 Which of the following countries along with
Russia was/were acceded as members of WTO
in December 2011?
(1) Montenegro (2) Samoa (3) Vanuatu
(a) 1, 2 & 3
(b) 1 & 2
(c) Only 1
(d) 1 & 3
41 The term “MESONET” is related to
(a) Mobile Phone
(b) Nuclear Energy
(c) Genetics
(d) Meteorology
42 Consider the following statements.
1. Pradhan Mantri Adarsh Gram Yojana
(PMAGY) is under Ministry of Social Justice
and Empowerment.
2. Pradhan Mantri Adarsh Gram Yojana
(PMAGY) is for the integrated development of
scheduled castes dominated villages
(a) Only 1 is true
(b) Only 2 is true
(c) Both 1 and 2 are true
(d) Neither 1 nor 2 is true
43 The only Indian bank in the Top 17 BRIC
banks, as listed by Global Finance
(a) ICICI
(b) State Bank of India
(c) Punjab National Bank
(d) Indian Overseas Bank
44 Who among the following is Cuba’s tourism
ambassador?
(a) Abishek Bachan
(b) Amitab Bachan
(c) Sharukh Khan
(d) Salman Khan
45 The Ministry of Health and Family Welfare
announced the launch of Weekly Iron and Folic
Supplementation (WIFS) programme which
will target
(a) Children
(b) Adolescents
(c) Pregnant women
(d) Both (a) and (c)
46 An elaborate commentary on Aryabhatiya
called the Aryabhatiya Bhasya was composed
by
(a) Nilakantha
Cracking IAS Study Circle, Chennai. PRELIMS 2012 SERIES
Current Affairs - Questions
_______________________________________________________
Address: - Y-block. 3020/1. 3rd
street. 13th
main road. Anna Nagar. Chennai-40
Landmark: Behind Natesan Institute of Co-operative Management on SHANTI COLONY ROAD.
Call: us: 09884554654. www.crackingIAS.com
(b) Baudhayana
(c) Brahmagupta
(d) Varahamihira
47 Where is Yoloten-Osman gas field?.
(a) Iran
(b) Turkmenistan
(c) Iraq
(d) Jordan
48 Which of the following is/are true about
Operation Cobra Gold 2012?
(a) It is military exercise sponsored by USA &
Thailand.
(b) India is a full participating nation in this
exercise.
(c) Both (a) and (b)
(d) Neither (a) nor (b)
49 Which of the following cities has earned the
sobriquet of `Orange City’?
(a) Salem
(b) Nagpur
(c) Pune
(d) None of these.
50 Which of the following is/are false w.r.t India
State of Forest Report 2011 released in
February 2012?
(a) The Forest and Tree cover is 23.81% of the
geographical area of the country.
(b) The state of Madhya Pradesh has the largest
forest cover in the country followed by Arunachal
Pradesh.
(c) Neither (a) Nor (b)
(d) Both (a) & (b)
51 “Lassa fever claims 13 lives in Nigeria” was the
recent headline news. Which of the following
is/are true in this context?.
(a) Lassa is a virus.
(b) It is transmitted primarily though contact with
rat excreta.
(c) Neither (a) Nor (b)
(d) Both (a) & (b)
52 'Janitor satellite' made to clean up space junk –
recent headline news. Which country has made
this?.
(a) Russia
(b) USA
(c) Switzerland
(d) China
53 Who was Ramkinkar Baij?
(a) Sculptor
(b) Violinist
(c) Environmental activist
(d) IT sector entrepreneur
54 Which of the following is/are true about
“Durban Platform for Enhanced Action,
December 2011” in news recently?.
(a) A new legally binding treaty to cut greenhouse
gases, expected to be decided by 2015 and come
into force by 2020.
(b) A goal of keeping global temperature
increases to “1.5 or 2.0 degrees C” above
preindustrial levels.
(c) Neither (a) Nor (b)
(d) Both (a) & (b)
55 In what context 'Sejeal stickers' were in news
recently?
(a) A stamp released on the eve of launch of
Project Seal to conserve seals
(b) A newly discovered high power adhesive
(c) An use and throw harmless tattoo
(d) None of these
56 Asia's first dolphin research centre is to come
up in
(a) Bihar
(b) West Bengal
(c) Chennai
(d) Goa
57 Sahyog-Kaijin is a joint exercise between
(a) India, Srilanka Coast Guards
(b) India, Japan Coast Guards
(c) India, Indonesia Coast Guards
(d) India, Australia Coast Guards
58 W.r.t. the Eighth Ministerial Conference of
WTO, 2011 which of the following is/are true?.
(a) India has strongly opposed a proposal by the
EU and a few other developed countries to include
new issues including energy & food security.
(b) India has ruled out freezing of custom duties at
the current levels.
(c) India has ruled out any dilution of the
flexibilities available under the WTO regime for
imposing export restrictions and taxes.
(d) 1, 2 & 3
Cracking IAS Study Circle, Chennai. PRELIMS 2012 SERIES
Current Affairs - Questions
_______________________________________________________
Address: - Y-block. 3020/1. 3rd
street. 13th
main road. Anna Nagar. Chennai-40
Landmark: Behind Natesan Institute of Co-operative Management on SHANTI COLONY ROAD.
Call: us: 09884554654. www.crackingIAS.com
59 First player to take 500 catches in Test cricket
(a) Mark Boucher
(b) Phillip Hughes
(c) Vernon Philander
(d) Sachin Tendulkar
60 Which of the following state has the lowest and
highest Infant Mortality respectively?.
(a) Goa & Madhya Pradesh
(b) Goa & Bihar
(c) Kerala & Orissa
(d) Kerala & Bihar
61 Which of the following Baltic country/(ies)
recently rejected Russian as official language?.
(a) Estonia
(b) Latvia
(c) Lithuania
(d) Both (a) and (b)
62 As of December 2011 New Parliamentary
Committee on welfare of Other Backward
Classes is to be constituted. Which of the
following is/are true w.r.t this?
(a) The Committee will comprise 30 members, 20
from Lok Sabha and 10 from Rajya Sabha.
(b) The members of the Committee shall hold
office for a period of one year from the date of the
first meeting and the quorum to constitute a sitting
shall be 10.
(c) Neither (a) Nor (b)
(d) Both (a) & (b)
63 What is “Stevia” which was in news recently?
(a) An indigenously made hydrogen fuelled car.
(b) A clone of Jersey cow recently made by
NDRI.
(c) A Cyclone which hit Philippines recently.
(d) None of these.
64 As of December 2011 how many of the states
are yet to constitute state HRCs?
(a) 6
(b) 8
(c) None
(d) Only One
65 Scientists have come up with a new wireless
device which they claim can detect the presence
of termites. Which of the following is/are true
in this regard?.
(a) It does so by "hearing" them chew through
timber.
(b) The device can communicate the termites'
GPS location.
(c) Neither (a) Nor (b)
(d) Both (a) & (b)
66 Which of the following is/are the components of
the Jawaharlal Nehru National Urban Renewal
Mission (JNNURM)?.
(1) Basic Services to the Urban Poor
(2) Urban Infrastructure and Governance
(3) Peer Experience and Reflective Learning
(a) Only 1
(b) Only 1 & 2
(c) 1, 2 & 3
(d) Only 1 & 3
67 “World's worst soccer violence in 15 years”.
This was the recent headline news. Where did
this happen?.
(a) Egypt
(b) Brazil
(c) Argentina
(d) Syria
68 The first ever vaccine to prevent kala azar, or
visceral leishmaniasis (VL), the world's second
largest parasitic killer after malaria, is
entering dual clinical trials in the
(a) US & Japan
(b) US & India
(c) India & Canada
(d) Japan & Canada
69 Global Wind Energy Council’s (GWEC)
annual statistics (2011) shows
(a) India had the most cumulative installed wind
power capacity.
(b) In second place was China in terms of
cumulative installed wind power capacity.
(c) Neither (a) Nor (b)
(d) Both (a) & (b)
70 “Scientists capture footage of rare whale” was
the recent headline news. Which of the
following is/are true in this context?.
(a) This was accomplished by The Australian
Antarctic Division team.
(b) These whales have dolphin-like beaks.
(c) Neither (a) Nor (b)
(d) Both (a) & (b)
Cracking IAS Study Circle, Chennai. PRELIMS 2012 SERIES
Current Affairs - Questions
_______________________________________________________
Address: - Y-block. 3020/1. 3rd
street. 13th
main road. Anna Nagar. Chennai-40
Landmark: Behind Natesan Institute of Co-operative Management on SHANTI COLONY ROAD.
Call: us: 09884554654. www.crackingIAS.com
71 How many destinations have been identified by
the Union Ministry of Tourism to be developed
as Mega Tourist Hubs across India (as of
January 2012)?
(a) 45
(b) 30
(c) 35
(d) 40
72 Bureau of Indian Standards is under
(a) Ministry of Consumer Affairs
(b) Ministry of Science & Technology
(c) Ministry of Commerce & Industry
(d) None of these
73 The Union budget has announced a provision
for the National Electricity Fund (NEF).
National Electricity Fund provides
(a) Interest subsidy for projects of electricity
distribution sector.
(b) Interest subsidy for projects of electricity
production sector.
(c) Interest subsidy for projects of solar energy
production sector.
(d) Interest subsidy for projects of eco friendly
energy production sector.
74 The figure here denotes a
songbird that weighs a mere
25 grams and completes the
longest known flight over
water of any songbird —
3,500 kilometres across the
Atlantic. It spends the
winter in Africa. Which are their breeding
grounds?
(a) Canadian Arctic
(b) Alaska
(c) Neither (a) Nor (b)
(d) Both (a) & (b)
75 Who heads Loomba Foundation which is
working for the welfare of widows?.
(a) Lakshmi Mittal
(b) Bill Gates
(c) Cherie Blair
(d) Indra Nooyi
CSAT (Paper 1 & 2)
2012 FINAL MOCK TEST
@ Chennai on
April 29, 2012
PAPER 1
Afternoon 2.30 p.m. - 4.30 p.m.
PAPER 2
April 30, 2012
Afternoon 2.30 p.m. - 4.30 p.m.
Prior Registration is Must.
Fees – Only Rs.50.
Call:SMS:9884554654
Cracking IAS Study Circle, Chennai. PRELIMS 2012 SERIES
Current Affairs - Questions
_______________________________________________________
Address: - Y-block. 3020/1. 3rd
street. 13th
main road. Anna Nagar. Chennai-40
Landmark: Behind Natesan Institute of Co-operative Management on SHANTI COLONY ROAD.
Call: us: 09884554654. www.crackingIAS.com
@ CHENNAI - Classroom Guidance
(Next Batch Classes for IAS 2013 & Mains 2012)
POLITICAL SCIENCE
& GS by Raja Sir & Team.
You cannot get a better guidance in Chennai.
Our Old students certify us.
I made a brilliant choice by joining
Cracking IAS Classes @ Chennai.
Arihant. Hyderabad. (In an interview to TV Channel)
Mains 2012 Test Series @ Chennai
Highly probable questions for MAINS 2012
Address
Cracking IAS, Y-block. 3020/1. 3rd street. 13th
main road. Anna Nagar. Chennai-40
Landmark: Behind Natesan Institute of Co-operative Management on Shanti Colony Road
500+ marks in GS Mains 2011 - Few examples (GS Mains 2010)
Paper 1 Q. 7(a) in – Our Mains current affairs notes P.no.176. Q. 8(c) in Paper 1 – Our Mains current affairs notes
P.no.200. Q. 9(f) & (g) in Paper 1 – Our Model test questions...& many more… Paper 2: Q. 2(a) & (c) – Our India and
World notes and also our model test question. Q. 3(a) & (d) – Our India and World notes. Q. 3(f) – Our Science and
Technology notes. Q. 3(h) – Our Mains current affairs notes P.no.136. Q. 5(b) – Our IAS Express August 2010 P.no.47.
Q. 5(d) – Our Science and Technology notes and also our model test question.. Q. 7(a) – Our Mains current affairs
Science and Technology notes P.no.1... & many more…Due to space constraints we could not list the other
questions… That’s because
We sweat 4 ur SWEET SUCCESS!
Excel with our Expertise in SOCIOLOGY
Many of our students scored 320+ in Sociology in Mains 2010 with
our member’s highest score being 357 in Mains 2010.
SOCIOLOGY by Raja Sir
Conquer the
corridors of power with our
Power Packed
Political Science
classes @ Chennai.
Our member scored 340 in
Political Science Mains. 190 in
Political Science Paper 2 - AIR
– 2 as per our known records
(IAS ‘07).
MAINS 2012 SURE SHOT Special
GS course
@ Chennai, From June 1st
week, 2012
An early bird Discount of 10% for those get admission
before May 10th
2012.
CSAT
By toppers in
Aptitude
Essay Enrichment Programme
@ Chennai from August, 2012
Call: us:
09884554554554554654
Cracking IAS Study Circle, Chennai. PRELIMS 2012 SERIES
Current Affairs - Questions
_______________________________________________________
Address: - Y-block. 3020/1. 3rd
street. 13th
main road. Anna Nagar. Chennai-40
Landmark: Behind Natesan Institute of Co-operative Management on SHANTI COLONY ROAD.
Call: us: 09884554654. www.crackingIAS.com
1 Under the concept of the Spice Park, Board will lease out the lands available in the Spice
Park to private entrepreneurs for developing their own processing plants for value
addition.
2nd
Spices park in India was established @ Jodhpur, Rajasthan.
A
2 • Headed by C. Rangarajan, Chairman of the Economic Advisory Council to the Prime
Minister, the Committee has been requested to complete its task as early as possible
and give its recommendations to the Prime Minister.
• The sugar industry has been demanding de-control of the sector on the plea that the
mandatory requirement of selling 10 per cent of production at below-cost rates for
supply to the ration shops is crippling it.
• Another key demand is to do away with the system under which the Food Ministry
decides on the quantity of the sugar that can be sold in the open market every month.
C
3 The Indian Bureau of Mines (IBM) is a subordinate office under the Ministry of Mines. It
is engaged in the promotion of scientific development of mineral resources of the country,
conservation of minerals, protection of environment in mines for minerals, other than coal,
petroleum and natural gas, atomic mineral and minor minerals. It performs regulatory
functions, namely enforcement of the Mineral Conservation and Development Rules,
1988, the relevant provisions of the Mines and Minerals (Development and Regulation)
Act, 1957, Mineral Concession Rules, 1960 and Environmental (Protection) Act, 1986 and
Rules made there under. It also undertakes scientific, techno-economic, research oriented
studies in various aspects of mining, geological studies, ore beneficiation and
environmental studies.
C
4 The State of Karnataka has set up a tiger protection force of fifty four foresters and forest
guards trained to protect the tigers its reserves. This is a first step of its kind ever taken in
India. The STPF has been constituted under the directive of MOEF this unit will be
initially in charge of Billgiri Rangaga Temple Wildlife Sanctuary, Bandipur Tiger Reserve
and Nagarhole Tiger Reserve in the state. The major threat to the tigers in Southern States
is from the infamous Khatni Gang active since last twenty five years in the country.
A
5 Prime Minister Manmohan Singh recently held talks with his Mauritian counterpart
Navinchandra Ramgoolam on the issue even as the two countries vowed to strengthen
security cooperation and to jointly battle the menace of piracy.
B
6 C
Cracking IAS Study Circle, Chennai. PRELIMS 2012 SERIES
Current Affairs - Questions
_______________________________________________________
Address: - Y-block. 3020/1. 3rd
street. 13th
main road. Anna Nagar. Chennai-40
Landmark: Behind Natesan Institute of Co-operative Management on SHANTI COLONY ROAD.
Call: us: 09884554654. www.crackingIAS.com
7 It is a Belgian scientific polar research station. A
8 D
9 Two Indian-origin men, including a journalist, are among South Africa's 10 heroes who
contributed to the public life in 2011. Journalist Yusuf Abramjee and philanthropist Dr
Imtiaz Sooliman joined the list, compiled by a newspaper, which was topped by South
African Public Protector Thuli Madonsela.
D
10 C
11 The completed Golden Quadrilateral will pass through 13 States of India:-
• Andhra Pradesh – 1,014 km, Uttar Pradesh, Rajasthan, Karnataka, Maharashtra,
Gujarat, Orissa, West Bengal, Tamil Nadu, Bihar, Jharkhand, Haryana, Delhi – 25 km.
The GQ project is managed by the National Highways Authority of India (NHAI)
under the Ministry of Road, Transport and Highways.
• The Mumbai-Pune Expressway, the first controlled-access toll road to be built in India
is a part of the GQ Project though not funded by NHAI, and separate from the main
highway. Infrastructure Leasing & Financial Services (IL&FS) has been one of the
major contributors to the infrastructural development activity in the GQ project.
C
12 C
13 India’s largest iron ore producer and exporter, NMDC Ltd, has announced the conclusion
of acquisition of a 50 per cent equity stake in Perth-based Australian iron ore, gold and
base metals exploration company Legacy Iron Ore Ltd for a consideration of A$18.89
million. According to a statement made to the stock exchanges, Legacy based in Western
Australia, is developing the Mt Bevan Iron Ore project, which has the potential to be one
of the largest resources of iron ore in Central Yilgarn region. The company is also
developing an advanced gold exploration project at Mt. Celia and holds other prospective
areas for iron and manganese and also gold at East Kimberley.
C
14 D
15 C
16 C
17 C
18 The Philippines' Puerto Princesa Underground River, the Amazon rainforest, Vietnam's
Halong Bay and Argentina's Iguazu Falls were named among the world's new seven
wonders of nature, according to organizers of a global poll. The other three crowned the
C
Cracking IAS Study Circle, Chennai. PRELIMS 2012 SERIES
Current Affairs - Questions
_______________________________________________________
Address: - Y-block. 3020/1. 3rd
street. 13th
main road. Anna Nagar. Chennai-40
Landmark: Behind Natesan Institute of Co-operative Management on SHANTI COLONY ROAD.
Call: us: 09884554654. www.crackingIAS.com
world's natural wonders are South Korea's Jeju Island, Indonesia's Komodo, and South
Africa's Table Mountain, said the New7Wonders foundation, citing provisional results.
19 National Girl Child Day is celebrated on 24 January every year since 2009. It was on this
date in 1966 Smt Indira Gandhi became the first woman Prime Minister of India. On
December 19, 2011, a historic milestone for girls' rights was achieved when the UN
General Assembly formally declared October 11 as the International Day of the Girl
Child.
B
20 Under this agreement, an advanced Remote Sensing satellite - SPOT-6 - weighing nearly
800 kg, built by ASTRIUM SAS, will be launched on-board ISRO's Polar Satellite Launch
Vehicle, during the second half of 2012.
In November 2010, under a commercial contract between ANTRIX and ASTRIUM, an
advanced communication satellite HYLAS was successfully built by ISRO and
ASTRIUM together for a European customer.
A
21 Pakistan defeated host Bangladesh to lift Asia Cup Cricket Trophy on 22 March 2012. C
22 Two pygmy marmosets, known as the world's smallest monkey species, have been
brought to Sakkarbaug Zoo in Junagadh under an exchange programme from Mysore Zoo.
A pigmy marmoset is just 5.5 to 6.5 inches 'tall', and weighs 300 to 500 grams. The
species is found in Western Brazil, South-Eastern Colombia, Eastern Ecuador and Eastern
Peru, zoo officials said, adding that the tiny animal's diet includes fruit, leaves, insects,
small reptiles and tree-sap.
A
23 D
24 • India has joined an exclusive club with the knowhow to process titanium sponge, a
key material in space and defence applications. ISRO has always depended on
overseas markets to source titanium sponge, which is created during the first stage of
processing titanium ore. It is the raw material used to make alloys that are used to
make rockets. The job of converting titanium sponge to titanium alloy will be done by
the Hyderabad-based Mishra Dhatu Nigam.
• Titanium ore is mined from beaches and the metal is known for its high strength but
low weight, making it an ideal material for aircraft manufacture, including fighter
aircraft. The material is also used in nuclear plants, and to make dental implants and
mobile phones.
• China dominates the production and use of titanium. Nine out of 18 companies
C
Cracking IAS Study Circle, Chennai. PRELIMS 2012 SERIES
Current Affairs - Questions
_______________________________________________________
Address: - Y-block. 3020/1. 3rd
street. 13th
main road. Anna Nagar. Chennai-40
Landmark: Behind Natesan Institute of Co-operative Management on SHANTI COLONY ROAD.
Call: us: 09884554654. www.crackingIAS.com
making titanium sponge are Chinese. The KMML plant not only helps India get parity
in strategic affairs but also opens the door for regional influence. In the pipeline is a
partnership to make titanium sponge with Kazakhstan. This is another opportunity for
India to expand its sphere of influence as a provider of special materials.
• Other research programmes undertaken by the KMML is also progressing. This
includes the novel “nano titanium pigment” which has tremendous scope in the
market. Its market price is ten times the price of titanium pigment, one of the current
products of KMML. The waste generated at the mineral separation unit of the KMML
is also being put to use.
• Titanium sponge is a porous form of titanium that is created during the first stage of
processing. In its natural form, titanium is widely available within the earth’s crust.
After being extracted, it is processed to remove excess materials and convert it into a
usable, although costly, product. The conversion process used with raw titanium is
called the KROLL PROCESS.
25 Country's first aerotropolis or airport city project at Andal in West Bengal will be
operational in the last quarter of 2012.
A
26 India had signed the Multilateral Convention on Mutual Administrative Assistance in Tax
Matters on 26th January, 2012. The Convention has been ratified by India by depositing
the Instrument of Ratification as on 21st February, 2012. By this, India has become the
first non-OECD, non-Council of Europe country to become a party to the Convention as
amended by the 2010 Protocol
A
27 It would prohibit companies that provide goods, services or credit worth USD 20 million
or more to Iran's energy industry from entering into or renewing state and local
government contracts.
B
28 An American teenager has become the youngest person to climb to the summit of the
highest mountains on each of the seven continents. Jordan Romero, 15, reached the
4,897m (16,067ft) summit of Vinson Massif in Antarctica on 24.12.2011, the final peak in
a quest he began six years ago.
'Seven summits'
Africa - Mt Kilimanjaro (5,892m; 19,340ft)
Antarctica - Vinson Massif (4,892; 16,050)
Australia - Mt Kosciuszko (2,228m; 7,310ft)
A
Cracking IAS Study Circle, Chennai. PRELIMS 2012 SERIES
Current Affairs - Questions
_______________________________________________________
Address: - Y-block. 3020/1. 3rd
street. 13th
main road. Anna Nagar. Chennai-40
Landmark: Behind Natesan Institute of Co-operative Management on SHANTI COLONY ROAD.
Call: us: 09884554654. www.crackingIAS.com
Oceania - Carstensz Pyramid (4,884m; 16,024ft)
Asia - Mt Everest (8,848m; 29,035ft)
Europe - Mt Elbrus (5,642m; 18,510ft)
North America - Mt McKinley (6,194m; 20,320ft)
South America - Aconcagua (6,962m; 22,841ft)
Highest mountains of each of the seven continents.
29 A
30 A
31 Auroras are a feature of the magnetosphere, the area surrounding a planet that is controlled
by its magnetic field and shaped by the solar wind, a steady flow of charged particles
emanating from the sun. Auroras are produced in the atmosphere as charged solar wind
particles accelerate in the magnetosphere and are guided by the magnetic field close to the
magnetic poles – that's why the Earthly auroras are found around high latitudes.
Auroras on Uranus are fainter than they are on Earth, and the planet is more than 4 billion
kilometers (2.5 billion miles) away. Previous Earth-bound attempts to detect the faint
auroras were inconclusive. Astronomers got their last good look at Uranian auroras 25
years ago when the Voyager 2 spacecraft whizzed past the planet and recorded spectra
from of the radiant display.
D
32 Accelerated pulses production programme (A3P) has been launched under the aegis of
National Food Security Mission (NFSM Pulses ), a Centrally-sponsored scheme.
D
33 A
34 She fought the Portuguese. She was also one of the earliest Indians to fight the colonial
powers and is sometimes regarded as the 'first woman freedom fighter of India'. Her story
is a legend in Karnataka, and is often enacted in Yakshagana, a popular folk theatre style
in Tulu Nadu.
A
35 Keshopur Chhamb Community Reserve in Gurdaspur District of Punjab, declared in 2007,
is the first Community Reserve in the country. Recently, Rajasthan government in India
declared "Jawai Bandh forests" as a conservation reserve forest. Jawai Bandh forest is
situated in Pali district and it is in close proximity of Kumbalgarh Sanctuary. (under
Section 36 of the amended Wildlife Protection Act, 1972). Located in the south-western
transition zone of the Aravallis on the Jawai river and the Luni river basin, and in close
proximity of Kumbalgarh sanctuary, the Jawai dam was built in 1957 for drinking water
purposes. Considered to be an ideal location for sighting crocodiles, the area also has a
C
Cracking IAS Study Circle, Chennai. PRELIMS 2012 SERIES
Current Affairs - Questions
_______________________________________________________
Address: - Y-block. 3020/1. 3rd
street. 13th
main road. Anna Nagar. Chennai-40
Landmark: Behind Natesan Institute of Co-operative Management on SHANTI COLONY ROAD.
Call: us: 09884554654. www.crackingIAS.com
large presence of turtles, fishes and aquatic birds. The Jawai Bandh terrain was declared a
closed area in 1983 and after the 2002 amendments it ceased to exist in that category.
36 D
37 Amid the escalating violence in Syria and a diplomatic outcry over Russia’s actions
blocking intervention by the United Nations, President Dmitri A. Medvedev bestowed a
prestigious cultural award on a Syrian writer Ali Ukla Ursan and poet who has publicly
applauded the terrorist attacks of Sept. 11, 2001, and expressed strong anti-Semitic views.
A
38 The 61st
Amendment act 1988 reduced the voting age from 21 years to 18 years for the
Lok Sabha and Assembly election. The Election commission has declared 25th
January,
its foundation day, as National Voters Day.
C
39 With an aim of hauling heavier trains, railways is ready to roll out a newly-designed 5500
horse power (HP) diesel locomotive equipped with all modern features including 100 km
per hour running speed. Aptly named as “Bheem", the high horse power locomotive is
currently undergoing safety trial. Manufactured at Diesel Locomotive Works at Varanasi,
the prototype has cost about Rs 17 crore to the railways. Designed jointly by the Railways
and EMD, a US-based company, the locomotive has a provision of toilet for crew, a first
in railways.
A
40 Russia acceded as a member, along with Samoa, Montenegro and Vanuatu. D
41 In meteorology, a mesonet is a network of automated weather stations designed to observe
mesoscale meteorological phenomena. The forecasting of the weather within the next six
hours is often referred to as nowcasting.
D
42 The PMAGY was announced in 2009-10 for integrated development of all villages with
more than 50 percent scheduled caste population. Presently, there are 44,000 such villages.
As a pilot project, it has been implemented in 1,000 villages in Assam, Bihar, Himachal
Pradesh, Rajasthan and Tamil Nadu with an allocation of Rs. 100 crore with each village
to get Rs. 10 lakh per year.
The Planning Commission has cleared the transfer of the Pradhan Mantri Adarsh Gram
Yojana (PMAGY) to the Ministry of Rural Development (MoRD) from the Ministry of
Social Justice and Empowerment. (MARCH 2012).
b
43 B
44 D
Cracking IAS Study Circle, Chennai. PRELIMS 2012 SERIES
Current Affairs - Questions
_______________________________________________________
Address: - Y-block. 3020/1. 3rd
street. 13th
main road. Anna Nagar. Chennai-40
Landmark: Behind Natesan Institute of Co-operative Management on SHANTI COLONY ROAD.
Call: us: 09884554654. www.crackingIAS.com
45 • The Ministry has suggested to the States that a fixed day in a week, preferably a
Monday, be earmarked as the day when Iron and Folic Acid tablet is provided to
adolescents.
• Funding for implementation of the scheme to the States would be provided under the
National Rural Health Mission.
• The States have been advised to project their fund requirements in the Programme
Implementation Plans for the year 2012-13 so that the scheme can be rolled out in the
forthcoming financial year.
• The key features of WIFS include administration of supervised Weekly Iron-folic Acid
Supplements of 100mg elemental iron and 500ug Folic acid; screening of target groups
for moderate/severe anaemia and referring these cases to an appropriate health facility;
Information and counselling for improving dietary intake and for taking actions for
prevention of intestinal worm infestation.
B
46 In this Bhasya, Nilakantha had discussed infinite series expansions of trigonometric
functions and problems of algebra and spherical geometry. Grahapareeksakrama is a
manual on making observations in astronomy based on instruments of the time.
A
47 The 1,735-km-long TAPI pipeline will run from Turkmenistan's Yoloten-Osman gas field
to Herat and Kandahar province of Afghanistan, before entering Pakistan. In Pakistan, it
will reach Multan via Quetta before ending at Fazilka (Punjab) in India.
India and Pakistan recently agreed in-principle to have a uniform transit fee for ferrying
natural gas through the proposed USD 7.6 billion pipeline from Turkmenistan. India will
pay a transit fee to Pakistan and Afghanistan for getting its share of 38 million standard
cubic metres per day of gas through the Turkmenistan-Afghanistan-Pakistan-India
pipeline, while Islamabad has to pay ferrying charges only to Afghanistan for allowing
passage of the fuel.
B
48 'Exercise Cobra Gold 2012' is designed to advance regional security by exercising a robust
multi-national force from nations sharing common goals and security commitments in the
Asia-Pacific region.
Full participating nations for Cobra Gold 12 include Thailand, the US, Singapore, Japan,
South Korea, Indonesia and Malaysia.
Several Countries which have been invited to participate on the multi-national planning
augmentation team included Australia, France, Canada, the United Kingdom, Bangladesh,
A
Cracking IAS Study Circle, Chennai. PRELIMS 2012 SERIES
Current Affairs - Questions
_______________________________________________________
Address: - Y-block. 3020/1. 3rd
street. 13th
main road. Anna Nagar. Chennai-40
Landmark: Behind Natesan Institute of Co-operative Management on SHANTI COLONY ROAD.
Call: us: 09884554654. www.crackingIAS.com
Italy, India, Nepal, Philippines and Vietnam
49 Nagpur lies precisely at the center of the country with the Zero Mile Marker indicating the
geographical center of India. Nagpur is also declared, "Tiger Capital of India" as it
connects many Tiger Reserves in India to the world.
B
50 In terms of percentage of forest cover in relation to total geographical area, Mizoram tops
with 90.68% followed by Lakshadweep with 84.56%. The India State of Forest Report
2011 contains the regular features like forest cover, tree cover, mangroves and growing
stock both in forests and areas outside forests. However, it adds three new chapters that
are of crucial importance in the present national and global worldview about forests. These
are: a detailed assessment of bamboo resources, a production-consumption assessment of
wood based on data stock in India’s forests reported under the NATCOM project.
C
51 Lassa fever is an acute viral hemorrhagic fever caused by the Lassa virus and first
described in 1969 in the town of Lassa, in Borno State, Nigeria, in the Yedseram river
valley at the south end of Lake Chad. The primary animal host of the Lassa virus is the
Natal Multimammate Mouse (Mastomys natalensis), an animal indigenous to most of Sub-
Saharan Africa. The virus is probably transmitted by contact with the feces or urine of
animals accessing grain stores in residences.
D
52 C
53 Ramkinkar Baij - the founding father of modern Indian sculpture. A
54 D
55 These stickers were found in the Bangkok house rented by the bungling Iranian bombers
has left Thai police puzzled. The word 'sejeal' is a reference to a miracle related in the
Quran, wherein birds pelted the enemies of the Prophet with baked clay 'stones' called
sejeals. Palestinians refer to their rockets and mortar shells as sejeal stones, and the
Islamic Republic of Iran makes a missle called 'sijjil'. Police in Thailand say three Iranian
nationals held in connection with a bomb blast in the Thai capital were plotting to attack
Israeli diplomats.
D
56 The man behind the proposal is R.K. Sinha, an expert on Gangetic river dolphins and
chairperson of the working group for dolphin conservation set up by the central
government. He said the centre was suggested by the Planning Commission and
subsequently received "in principle" approval by the state government. The Vikramshila
Gangetic Dolphin Sanctuary, India's only dolphin sanctuary, spread over 50 km along
the Ganges, is located in Bihar's Bhagalpur district. The Gangetic river dolphin is India's
A
Cracking IAS Study Circle, Chennai. PRELIMS 2012 SERIES
Current Affairs - Questions
_______________________________________________________
Address: - Y-block. 3020/1. 3rd
street. 13th
main road. Anna Nagar. Chennai-40
Landmark: Behind Natesan Institute of Co-operative Management on SHANTI COLONY ROAD.
Call: us: 09884554654. www.crackingIAS.com
national aquatic animal but frequently falls prey to poachers. Their carcasses are found
regularly on river banks.
Gangetic river dolphins fall under Schedule I of the Indian Wildlife (Protection) Act and
have been declared an endangered species by the International Union for Conservation of
Nature (IUCN).
The Gangetic river dolphin is one of the four freshwater dolphin species in the world. The
other three are found in the Yangtze river in China, the Indus river in Pakistan and the
Amazon river in South America.
The Gangetic river species - found in India, Bangladesh and Nepal - is blind and finds its
way and prey in the river waters through 'echoes'.
57 B
58 G33 countries is a coalition of agricultural economies with defensive interests in
agriculture, coordinated by Indonesia.
D
59 Mark Boucher, the South African wicketkeeper, has become the first cricketer to take 500
Test catches during the opening session of the first Test against Australia on Wednesday.
His 500th dismissal was that of Australian opener Phillip Hughes who was caught behind
off the bowling of Test debutant Vernon Philander.
A
60 Goa has the lowest IMR of 10 infant deaths followed by Kerala with 13 infant deaths per
1,000 live births. Madhya Pradesh has the highest IMR of 62 infant deaths per 1,000,
followed by Uttar Pradesh and Odisha.
A
61 The term Baltic states - Estonia, Latvia, Lithuania (from north to south). B
62 The Committee will consider the reports submitted by the National Commission for
Backward Classes and report to both the Houses on the measures that should be taken by
the Union government within its purview and the Administrations of the Union
Territories.
D
63 Stevia leaves contain stevioside, which is 300 times sweeter than sugar. Artificial
sweeteners have been mired in controversy as some of them have been found to be
carcinogenic. On July 26, Food Safety and Standards Authority of India (FSSAI) issued a
notification clarifying that stevia is not a food item. However, currently a scientific panel
in FSSAI is considering whether stevia can be approved as a food item.
A
64 States which have not constituted the state human rights commissions are Arunachal
Pradesh, Goa, Haryana, Meghalaya, Mizoram, Nagaland, Tripura and Uttarakhand.
B
65 A team at Edith Cowan University says that the sensitive device, called wireless smart D
Cracking IAS Study Circle, Chennai. PRELIMS 2012 SERIES
Current Affairs - Questions
_______________________________________________________
Address: - Y-block. 3020/1. 3rd
street. 13th
main road. Anna Nagar. Chennai-40
Landmark: Behind Natesan Institute of Co-operative Management on SHANTI COLONY ROAD.
Call: us: 09884554654. www.crackingIAS.com
probe (WiSPr) network for acoustic detection, is also capable of locating termite activity
in timber bridges and wooden power poles.
66 "Peer Experience and Reflective Learning" (PEARL) is an initiative under JNNURM to
support cities to actively pursue activities in implementation of projects and reforms.
C
67 A
68 The trials funded by the Bill & Melinda Gates Foundation are launched by the Infectious
Disease Research Institute (IDRI), a Seattle-based non-profit research body with a
companion Phase 1 trial planned in India, an epicenter of the disease. IDRI is transferring
its technology to Pune's Gennova Biopharmaceuticals, which has opened a new facility
that will produce the candidate vaccine for the clinical trial to begin late in 2012. The
phase-1 trial in the US has been launched in Washington state in the US. The IDRI
vaccine, known as LEISH-F3 + GLA-SE, is a highly purified, recombinant vaccine. It
incorporates two fused Leishmania parasite proteins and a powerful adjuvant to stimulate
an immune response against the parasite.
B
69 China had the most cumulative installed wind power capacity — 62,733 MW — at the end
of 2011, followed by the United States with 46,919 MW. Germany was in third place with
29,060 MW followed by Spain with 21,674 MW. In fifth place was India with 16,084
MW.
C
70 D
71 For each identified mega destination and circuit, the Ministry of Tourism contribution as
central financial assistance is capped at Rs.25.00 crore and Rs.50.00 crore respectively
under the scheme of “Product/Infrastructure Development for Destinations and Circuits”.
A
72 A
73 The fund is being set up to provide interest subsidy on loans to be disbursed to the
Distribution Companies (DISCOMS) - both in the public and private sector, to improve
the distribution network for areas not covered by Rajiv Gandhi Gramin Vidyutikaran
Yojana (RGGVY) and Restructured Accelerated Power Development and Reforms
Programme (R-APDRP) project areas. For the first time, India is planning a mandatory
rating system for 65 state-owned distribution firms. Once the government assigns ratings
to SEBs, they will form the basis on which state-controlled banks and financial institutions
will lend to them. If a utility is highly rated, it will be eligible for more funds at a lower
rate of interest.
The Centre has set up the fund following recommendation of the V K Shunglu panel.
A
Cracking IAS Study Circle, Chennai. PRELIMS 2012 SERIES
Current Affairs - Questions
_______________________________________________________
Address: - Y-block. 3020/1. 3rd
street. 13th
main road. Anna Nagar. Chennai-40
Landmark: Behind Natesan Institute of Co-operative Management on SHANTI COLONY ROAD.
Call: us: 09884554654. www.crackingIAS.com
The panel recommended that the states be asked to take over distressed loans of discoms if
they want to avail financial assistance from the Centre. Besides, they should also commit
on implementation of power reforms in a time bound manner while negotiating terms of
availing central financial package.
74 For the first time, Ontario scientists have proven what they long suspected — the wheatear
is the only songbird that breeds in the Canadian Arctic and Alaska and spends the winter
in Africa. The migration can take from one to three months.
They also showed that the wheatear completes the longest known flight over water of any
songbird — 3,500 kilometres across the Atlantic. That remarkable passage takes about
four days of non-stop flying.
There are two separate populations of wheatears in the far north. One breeds in Alaska,
flies west over Russia, Kazakhstan and the Arabian Desert some 14,500 km, ending up in
Sudan and Uganda in East Africa. Another, which breeds in the Canadian Arctic near
Iqaluit heads the opposite direction, east over the Atlantic, stopping for the winter in
Mauritania in West Africa, a journey of 7,500 km. The species is of particular interest to
scientists, because it has one of the largest ranges of any songbird in the world.
D
75 In 2005, Loomba Foundation president Cherie Blair launched International Widows Day
at the House of Lords in London and over the next five years, the Foundation campaigned
for international recognition of this day as a focus for sustained, effective, global action to
bring about a radical and lasting transformation in the plight of widows. The significance
of 23 June is that this is the day, in 1954, that the woman who inspired the founding of the
Loomba Foundation, Shrimati Pushpa Wati Loomba, became a widow.
The international goodwill ambassador for The Loomba Foundation – Kajol.
C

More Related Content

Viewers also liked

Viewers also liked (6)

Raa
RaaRaa
Raa
 
Desafios ja 2012
Desafios ja 2012Desafios ja 2012
Desafios ja 2012
 
Sonica
SonicaSonica
Sonica
 
Important moments of my life
Important moments of my lifeImportant moments of my life
Important moments of my life
 
A different kind_of_guitar_company
A different kind_of_guitar_companyA different kind_of_guitar_company
A different kind_of_guitar_company
 
RPP PRAKARYA KURTILAS_MTs. NEGERI KASOMALANG
RPP PRAKARYA KURTILAS_MTs. NEGERI KASOMALANGRPP PRAKARYA KURTILAS_MTs. NEGERI KASOMALANG
RPP PRAKARYA KURTILAS_MTs. NEGERI KASOMALANG
 

Similar to Current affairs april issue magazine

IAS CSAT 2018 Paper 1 - Questions With Solutions || Sourav Sir's Classes
IAS CSAT 2018 Paper 1 - Questions With Solutions || Sourav Sir's ClassesIAS CSAT 2018 Paper 1 - Questions With Solutions || Sourav Sir's Classes
IAS CSAT 2018 Paper 1 - Questions With Solutions || Sourav Sir's ClassesSOURAV DAS
 
Ias csat paper 1 full solve 2018 SOURAV SIR'S CLASSES 9836793076
Ias csat paper 1 full solve 2018 SOURAV SIR'S CLASSES  9836793076Ias csat paper 1 full solve 2018 SOURAV SIR'S CLASSES  9836793076
Ias csat paper 1 full solve 2018 SOURAV SIR'S CLASSES 9836793076SOURAV DAS
 
Indian Administrative Service(IAS) CSAT PAPER I | STUDY MATERIALS | NOTES SOL...
Indian Administrative Service(IAS) CSAT PAPER I | STUDY MATERIALS | NOTES SOL...Indian Administrative Service(IAS) CSAT PAPER I | STUDY MATERIALS | NOTES SOL...
Indian Administrative Service(IAS) CSAT PAPER I | STUDY MATERIALS | NOTES SOL...SOURAV DAS
 
11. PPT-April 2020.ppt
11. PPT-April 2020.ppt11. PPT-April 2020.ppt
11. PPT-April 2020.pptRamLalop
 
Current Affairs Quiz Pdf (september 2015) By DaytoDayGk.com
Current Affairs Quiz Pdf (september 2015) By DaytoDayGk.comCurrent Affairs Quiz Pdf (september 2015) By DaytoDayGk.com
Current Affairs Quiz Pdf (september 2015) By DaytoDayGk.comDaytodayGk
 
Apsc test series 3 , 2017
Apsc test series 3 , 2017Apsc test series 3 , 2017
Apsc test series 3 , 2017jitendra taid
 
Upsc ias pre 2015 gs answer key
Upsc ias pre 2015 gs answer keyUpsc ias pre 2015 gs answer key
Upsc ias pre 2015 gs answer keyMaan Chaudhary
 
10. PPT-March 2020.pptx
10. PPT-March 2020.pptx10. PPT-March 2020.pptx
10. PPT-March 2020.pptxRamLalop
 
1 allahabad bank-probationary-officer-exam-general-awareness-solved-paper-2007
1 allahabad bank-probationary-officer-exam-general-awareness-solved-paper-20071 allahabad bank-probationary-officer-exam-general-awareness-solved-paper-2007
1 allahabad bank-probationary-officer-exam-general-awareness-solved-paper-2007Kumar Nirmal Prasad
 
1 allahabad bank-probationary-officer-exam-general-awareness-solved-paper-2007
1 allahabad bank-probationary-officer-exam-general-awareness-solved-paper-20071 allahabad bank-probationary-officer-exam-general-awareness-solved-paper-2007
1 allahabad bank-probationary-officer-exam-general-awareness-solved-paper-2007Kumar Nirmal Prasad
 
Upsc 2017 solutions
Upsc 2017 solutionsUpsc 2017 solutions
Upsc 2017 solutionsRahul Sharma
 
5070_LST-PPT-March-2022.pptx
5070_LST-PPT-March-2022.pptx5070_LST-PPT-March-2022.pptx
5070_LST-PPT-March-2022.pptxssuser4eceba1
 
120 ca mc qs 2
120 ca mc qs   2120 ca mc qs   2
120 ca mc qs 2ashi004
 
Yearly current affairs quiz (2020)
Yearly current affairs quiz (2020)Yearly current affairs quiz (2020)
Yearly current affairs quiz (2020)Takshila Learning
 

Similar to Current affairs april issue magazine (20)

IAS CSAT 2018 Paper 1 - Questions With Solutions || Sourav Sir's Classes
IAS CSAT 2018 Paper 1 - Questions With Solutions || Sourav Sir's ClassesIAS CSAT 2018 Paper 1 - Questions With Solutions || Sourav Sir's Classes
IAS CSAT 2018 Paper 1 - Questions With Solutions || Sourav Sir's Classes
 
Ias csat paper 1 full solve 2018 SOURAV SIR'S CLASSES 9836793076
Ias csat paper 1 full solve 2018 SOURAV SIR'S CLASSES  9836793076Ias csat paper 1 full solve 2018 SOURAV SIR'S CLASSES  9836793076
Ias csat paper 1 full solve 2018 SOURAV SIR'S CLASSES 9836793076
 
Indian Administrative Service(IAS) CSAT PAPER I | STUDY MATERIALS | NOTES SOL...
Indian Administrative Service(IAS) CSAT PAPER I | STUDY MATERIALS | NOTES SOL...Indian Administrative Service(IAS) CSAT PAPER I | STUDY MATERIALS | NOTES SOL...
Indian Administrative Service(IAS) CSAT PAPER I | STUDY MATERIALS | NOTES SOL...
 
11. PPT-April 2020.ppt
11. PPT-April 2020.ppt11. PPT-April 2020.ppt
11. PPT-April 2020.ppt
 
Current Affairs Quiz Pdf (september 2015) By DaytoDayGk.com
Current Affairs Quiz Pdf (september 2015) By DaytoDayGk.comCurrent Affairs Quiz Pdf (september 2015) By DaytoDayGk.com
Current Affairs Quiz Pdf (september 2015) By DaytoDayGk.com
 
Apsc test series 3 , 2017
Apsc test series 3 , 2017Apsc test series 3 , 2017
Apsc test series 3 , 2017
 
Upsc ias pre 2015 gs answer key
Upsc ias pre 2015 gs answer keyUpsc ias pre 2015 gs answer key
Upsc ias pre 2015 gs answer key
 
CURRENT AFFAIRS-2023.pdf
CURRENT AFFAIRS-2023.pdfCURRENT AFFAIRS-2023.pdf
CURRENT AFFAIRS-2023.pdf
 
Naren Quiz 39 Dr.K.Karthikeyan
Naren Quiz 39 Dr.K.KarthikeyanNaren Quiz 39 Dr.K.Karthikeyan
Naren Quiz 39 Dr.K.Karthikeyan
 
10. PPT-March 2020.pptx
10. PPT-March 2020.pptx10. PPT-March 2020.pptx
10. PPT-March 2020.pptx
 
Naren Quiz 21 Dr.K.Karthikeyan
Naren Quiz 21 Dr.K.KarthikeyanNaren Quiz 21 Dr.K.Karthikeyan
Naren Quiz 21 Dr.K.Karthikeyan
 
Khan
KhanKhan
Khan
 
Naren Quiz 37 Dr.K.Karthikeyan
Naren Quiz 37 Dr.K.KarthikeyanNaren Quiz 37 Dr.K.Karthikeyan
Naren Quiz 37 Dr.K.Karthikeyan
 
1 allahabad bank-probationary-officer-exam-general-awareness-solved-paper-2007
1 allahabad bank-probationary-officer-exam-general-awareness-solved-paper-20071 allahabad bank-probationary-officer-exam-general-awareness-solved-paper-2007
1 allahabad bank-probationary-officer-exam-general-awareness-solved-paper-2007
 
1 allahabad bank-probationary-officer-exam-general-awareness-solved-paper-2007
1 allahabad bank-probationary-officer-exam-general-awareness-solved-paper-20071 allahabad bank-probationary-officer-exam-general-awareness-solved-paper-2007
1 allahabad bank-probationary-officer-exam-general-awareness-solved-paper-2007
 
Upsc 2017 solutions
Upsc 2017 solutionsUpsc 2017 solutions
Upsc 2017 solutions
 
Naren Quiz 31 Dr.K.Karthikeyan
Naren Quiz 31 Dr.K.KarthikeyanNaren Quiz 31 Dr.K.Karthikeyan
Naren Quiz 31 Dr.K.Karthikeyan
 
5070_LST-PPT-March-2022.pptx
5070_LST-PPT-March-2022.pptx5070_LST-PPT-March-2022.pptx
5070_LST-PPT-March-2022.pptx
 
120 ca mc qs 2
120 ca mc qs   2120 ca mc qs   2
120 ca mc qs 2
 
Yearly current affairs quiz (2020)
Yearly current affairs quiz (2020)Yearly current affairs quiz (2020)
Yearly current affairs quiz (2020)
 

Recently uploaded

ClimART Action | eTwinning Project
ClimART Action    |    eTwinning ProjectClimART Action    |    eTwinning Project
ClimART Action | eTwinning Projectjordimapav
 
4.9.24 School Desegregation in Boston.pptx
4.9.24 School Desegregation in Boston.pptx4.9.24 School Desegregation in Boston.pptx
4.9.24 School Desegregation in Boston.pptxmary850239
 
Grade 9 Quarter 4 Dll Grade 9 Quarter 4 DLL.pdf
Grade 9 Quarter 4 Dll Grade 9 Quarter 4 DLL.pdfGrade 9 Quarter 4 Dll Grade 9 Quarter 4 DLL.pdf
Grade 9 Quarter 4 Dll Grade 9 Quarter 4 DLL.pdfJemuel Francisco
 
Expanded definition: technical and operational
Expanded definition: technical and operationalExpanded definition: technical and operational
Expanded definition: technical and operationalssuser3e220a
 
Transaction Management in Database Management System
Transaction Management in Database Management SystemTransaction Management in Database Management System
Transaction Management in Database Management SystemChristalin Nelson
 
Team Lead Succeed – Helping you and your team achieve high-performance teamwo...
Team Lead Succeed – Helping you and your team achieve high-performance teamwo...Team Lead Succeed – Helping you and your team achieve high-performance teamwo...
Team Lead Succeed – Helping you and your team achieve high-performance teamwo...Association for Project Management
 
Grade Three -ELLNA-REVIEWER-ENGLISH.pptx
Grade Three -ELLNA-REVIEWER-ENGLISH.pptxGrade Three -ELLNA-REVIEWER-ENGLISH.pptx
Grade Three -ELLNA-REVIEWER-ENGLISH.pptxkarenfajardo43
 
ICS 2208 Lecture Slide Notes for Topic 6
ICS 2208 Lecture Slide Notes for Topic 6ICS 2208 Lecture Slide Notes for Topic 6
ICS 2208 Lecture Slide Notes for Topic 6Vanessa Camilleri
 
4.16.24 21st Century Movements for Black Lives.pptx
4.16.24 21st Century Movements for Black Lives.pptx4.16.24 21st Century Movements for Black Lives.pptx
4.16.24 21st Century Movements for Black Lives.pptxmary850239
 
MS4 level being good citizen -imperative- (1) (1).pdf
MS4 level   being good citizen -imperative- (1) (1).pdfMS4 level   being good citizen -imperative- (1) (1).pdf
MS4 level being good citizen -imperative- (1) (1).pdfMr Bounab Samir
 
31 ĐỀ THI THỬ VÀO LỚP 10 - TIẾNG ANH - FORM MỚI 2025 - 40 CÂU HỎI - BÙI VĂN V...
31 ĐỀ THI THỬ VÀO LỚP 10 - TIẾNG ANH - FORM MỚI 2025 - 40 CÂU HỎI - BÙI VĂN V...31 ĐỀ THI THỬ VÀO LỚP 10 - TIẾNG ANH - FORM MỚI 2025 - 40 CÂU HỎI - BÙI VĂN V...
31 ĐỀ THI THỬ VÀO LỚP 10 - TIẾNG ANH - FORM MỚI 2025 - 40 CÂU HỎI - BÙI VĂN V...Nguyen Thanh Tu Collection
 
Congestive Cardiac Failure..presentation
Congestive Cardiac Failure..presentationCongestive Cardiac Failure..presentation
Congestive Cardiac Failure..presentationdeepaannamalai16
 
ARTERIAL BLOOD GAS ANALYSIS........pptx
ARTERIAL BLOOD  GAS ANALYSIS........pptxARTERIAL BLOOD  GAS ANALYSIS........pptx
ARTERIAL BLOOD GAS ANALYSIS........pptxAneriPatwari
 
BIOCHEMISTRY-CARBOHYDRATE METABOLISM CHAPTER 2.pptx
BIOCHEMISTRY-CARBOHYDRATE METABOLISM CHAPTER 2.pptxBIOCHEMISTRY-CARBOHYDRATE METABOLISM CHAPTER 2.pptx
BIOCHEMISTRY-CARBOHYDRATE METABOLISM CHAPTER 2.pptxSayali Powar
 
Active Learning Strategies (in short ALS).pdf
Active Learning Strategies (in short ALS).pdfActive Learning Strategies (in short ALS).pdf
Active Learning Strategies (in short ALS).pdfPatidar M
 
Daily Lesson Plan in Mathematics Quarter 4
Daily Lesson Plan in Mathematics Quarter 4Daily Lesson Plan in Mathematics Quarter 4
Daily Lesson Plan in Mathematics Quarter 4JOYLYNSAMANIEGO
 
ICS2208 Lecture6 Notes for SL spaces.pdf
ICS2208 Lecture6 Notes for SL spaces.pdfICS2208 Lecture6 Notes for SL spaces.pdf
ICS2208 Lecture6 Notes for SL spaces.pdfVanessa Camilleri
 
ESP 4-EDITED.pdfmmcncncncmcmmnmnmncnmncmnnjvnnv
ESP 4-EDITED.pdfmmcncncncmcmmnmnmncnmncmnnjvnnvESP 4-EDITED.pdfmmcncncncmcmmnmnmncnmncmnnjvnnv
ESP 4-EDITED.pdfmmcncncncmcmmnmnmncnmncmnnjvnnvRicaMaeCastro1
 
DIFFERENT BASKETRY IN THE PHILIPPINES PPT.pptx
DIFFERENT BASKETRY IN THE PHILIPPINES PPT.pptxDIFFERENT BASKETRY IN THE PHILIPPINES PPT.pptx
DIFFERENT BASKETRY IN THE PHILIPPINES PPT.pptxMichelleTuguinay1
 

Recently uploaded (20)

Mattingly "AI & Prompt Design: Large Language Models"
Mattingly "AI & Prompt Design: Large Language Models"Mattingly "AI & Prompt Design: Large Language Models"
Mattingly "AI & Prompt Design: Large Language Models"
 
ClimART Action | eTwinning Project
ClimART Action    |    eTwinning ProjectClimART Action    |    eTwinning Project
ClimART Action | eTwinning Project
 
4.9.24 School Desegregation in Boston.pptx
4.9.24 School Desegregation in Boston.pptx4.9.24 School Desegregation in Boston.pptx
4.9.24 School Desegregation in Boston.pptx
 
Grade 9 Quarter 4 Dll Grade 9 Quarter 4 DLL.pdf
Grade 9 Quarter 4 Dll Grade 9 Quarter 4 DLL.pdfGrade 9 Quarter 4 Dll Grade 9 Quarter 4 DLL.pdf
Grade 9 Quarter 4 Dll Grade 9 Quarter 4 DLL.pdf
 
Expanded definition: technical and operational
Expanded definition: technical and operationalExpanded definition: technical and operational
Expanded definition: technical and operational
 
Transaction Management in Database Management System
Transaction Management in Database Management SystemTransaction Management in Database Management System
Transaction Management in Database Management System
 
Team Lead Succeed – Helping you and your team achieve high-performance teamwo...
Team Lead Succeed – Helping you and your team achieve high-performance teamwo...Team Lead Succeed – Helping you and your team achieve high-performance teamwo...
Team Lead Succeed – Helping you and your team achieve high-performance teamwo...
 
Grade Three -ELLNA-REVIEWER-ENGLISH.pptx
Grade Three -ELLNA-REVIEWER-ENGLISH.pptxGrade Three -ELLNA-REVIEWER-ENGLISH.pptx
Grade Three -ELLNA-REVIEWER-ENGLISH.pptx
 
ICS 2208 Lecture Slide Notes for Topic 6
ICS 2208 Lecture Slide Notes for Topic 6ICS 2208 Lecture Slide Notes for Topic 6
ICS 2208 Lecture Slide Notes for Topic 6
 
4.16.24 21st Century Movements for Black Lives.pptx
4.16.24 21st Century Movements for Black Lives.pptx4.16.24 21st Century Movements for Black Lives.pptx
4.16.24 21st Century Movements for Black Lives.pptx
 
MS4 level being good citizen -imperative- (1) (1).pdf
MS4 level   being good citizen -imperative- (1) (1).pdfMS4 level   being good citizen -imperative- (1) (1).pdf
MS4 level being good citizen -imperative- (1) (1).pdf
 
31 ĐỀ THI THỬ VÀO LỚP 10 - TIẾNG ANH - FORM MỚI 2025 - 40 CÂU HỎI - BÙI VĂN V...
31 ĐỀ THI THỬ VÀO LỚP 10 - TIẾNG ANH - FORM MỚI 2025 - 40 CÂU HỎI - BÙI VĂN V...31 ĐỀ THI THỬ VÀO LỚP 10 - TIẾNG ANH - FORM MỚI 2025 - 40 CÂU HỎI - BÙI VĂN V...
31 ĐỀ THI THỬ VÀO LỚP 10 - TIẾNG ANH - FORM MỚI 2025 - 40 CÂU HỎI - BÙI VĂN V...
 
Congestive Cardiac Failure..presentation
Congestive Cardiac Failure..presentationCongestive Cardiac Failure..presentation
Congestive Cardiac Failure..presentation
 
ARTERIAL BLOOD GAS ANALYSIS........pptx
ARTERIAL BLOOD  GAS ANALYSIS........pptxARTERIAL BLOOD  GAS ANALYSIS........pptx
ARTERIAL BLOOD GAS ANALYSIS........pptx
 
BIOCHEMISTRY-CARBOHYDRATE METABOLISM CHAPTER 2.pptx
BIOCHEMISTRY-CARBOHYDRATE METABOLISM CHAPTER 2.pptxBIOCHEMISTRY-CARBOHYDRATE METABOLISM CHAPTER 2.pptx
BIOCHEMISTRY-CARBOHYDRATE METABOLISM CHAPTER 2.pptx
 
Active Learning Strategies (in short ALS).pdf
Active Learning Strategies (in short ALS).pdfActive Learning Strategies (in short ALS).pdf
Active Learning Strategies (in short ALS).pdf
 
Daily Lesson Plan in Mathematics Quarter 4
Daily Lesson Plan in Mathematics Quarter 4Daily Lesson Plan in Mathematics Quarter 4
Daily Lesson Plan in Mathematics Quarter 4
 
ICS2208 Lecture6 Notes for SL spaces.pdf
ICS2208 Lecture6 Notes for SL spaces.pdfICS2208 Lecture6 Notes for SL spaces.pdf
ICS2208 Lecture6 Notes for SL spaces.pdf
 
ESP 4-EDITED.pdfmmcncncncmcmmnmnmncnmncmnnjvnnv
ESP 4-EDITED.pdfmmcncncncmcmmnmnmncnmncmnnjvnnvESP 4-EDITED.pdfmmcncncncmcmmnmnmncnmncmnnjvnnv
ESP 4-EDITED.pdfmmcncncncmcmmnmnmncnmncmnnjvnnv
 
DIFFERENT BASKETRY IN THE PHILIPPINES PPT.pptx
DIFFERENT BASKETRY IN THE PHILIPPINES PPT.pptxDIFFERENT BASKETRY IN THE PHILIPPINES PPT.pptx
DIFFERENT BASKETRY IN THE PHILIPPINES PPT.pptx
 

Current affairs april issue magazine

  • 1. Cracking IAS Study Circle, Chennai. PRELIMS 2012 SERIES Current Affairs - Questions _______________________________________________________ Address: - Y-block. 3020/1. 3rd street. 13th main road. Anna Nagar. Chennai-40 Landmark: Behind Natesan Institute of Co-operative Management on SHANTI COLONY ROAD. Call: us: 09884554654. www.crackingIAS.com 1 Consider the following statements. 1. First Spices park in India was established @ Chhindwara, Madhya Pradesh under the ASIDE scheme. 2. Under the concept of the Spice Park, Spices Board will lease out the lands available in the Spice Park to PSU’s for developing their own processing plants for value addition. (a) Only 1 is true (b) Only 2 is true (c) Both 1 and 2 are true (d) Neither 1 nor 2 is true 2 Who heads the heads expert committee on sugar sector? (a) Kaushik Basu (b) Ashok Gulati (c) C. Rangarajan (d) None of these 3 Minerals Resource Book (MRB) is prepared by the (a) Ministry of Earth Sciences. (b) Geological Survey of India (GSI). (c) Indian Bureau of Mines (IBM). (d) Ministry of Trade and Commerce. 4 Which is the first state in India to deploy the Special Tiger Protection Force (STPF)?. (a) Karnataka (b) Madhya Pradesh (c) Tamilnadu (d) West Bengal 5 India gifted a helicopter in November 2009 as part of an economic package and installed a coastal surveillance radar system in April 2011, also recently in early 2012 announced a new line of credit of US$250 million and a grant of US$ 20 million to a country. Which is the country cited under reference?. (a) Maldives (b) Mauritius (c) Nigeria (d) Zimbabwe 6 Consider the following statements. 1. Fertilizer is currently the main item of export from Belarus to India. 2. Potash is the main import item of India from Belarus. (a) Only 1 is true (b) Only 2 is true (c) Both 1 and 2 are true (d) Neither 1 nor 2 is true 7 Which of the following is/are true about “Princess Elisabeth Antarctica” in news recently? (a) It is conceived as a zero-emission station, entirely powered by renewable energy. (b) It is an Australian scientific polar research station. (c) Neither (a) Nor (b) (d) Both (a) & (b) 8 Concerns over the effects of Tributyltin (TBT) are often in news. Which of the following is/are true w.r.t. Tributyltin (TBT)?. (a) It is an ingredient in certain antifouling paints used on ships. (b) It is present in Disposable diapers. (c) It causes hormonal problems. (d) All the above. 9 Who authored “Glimpses of Indian Women Hockey”? (a) Ajay Maken (b) Leandro Negre (c) V.N.Balan (d) K. Arumugam 10 Bangladesh and India formally began signing of strip-maps of 4,156kms common boundary to settle the long-running issue between the two close door neighbours. Which of the following is/are not the feature of “Strip Map”? (a) It is an un-scaled drawing of a route, includes critical points along the border. (b) It usually incorporates distances, route-side features and town facilities on a simple flip-over style map. (c) Neither (a) Nor (b) (d) Both (a) & (b) 11 Which of the following is/are false w.r.t. Golden Quadrilateral?. (a) Only National Highways are used in the Golden Quadrilateral. (b) The present status of Golden Quadrilateral project is complete. (c) Neither (a) Nor (b) (d) Both (a) & (b) 12 Consider the following statements.
  • 2. Cracking IAS Study Circle, Chennai. PRELIMS 2012 SERIES Current Affairs - Questions _______________________________________________________ Address: - Y-block. 3020/1. 3rd street. 13th main road. Anna Nagar. Chennai-40 Landmark: Behind Natesan Institute of Co-operative Management on SHANTI COLONY ROAD. Call: us: 09884554654. www.crackingIAS.com 1. The Mid Day Meal Scheme is the largest noon meal scheme in the world. 2. Mid-day meal Scheme tries to legitimize the directive principles enshrined in our constitution. (a) Only 1 is true (b) Only 2 is true (c) Both 1 and 2 are true (d) Neither 1 nor 2 is true 13 Mt Bevan iron ore Project is based at (a) Canada (b) USA (c) Australia (d) Germany 14 Which of the following Asian country has recently set up a laboratory to conduct research on medicines for astronauts during space missions?. (a) India (b) Pakistan (c) Indonesia (d) China 15 Which of the following is/are not the feature of The National Common Mobility Card (NCMC) named 'More', which was unveiled by urban development ministry?. (a) One can also pay parking and toll fees using this card. (b) The card, once it becomes operational, can be used across different modes of transport like buses, Metro, Bus Rapid Transport System etc. (c) Neither (a) Nor (b) (d) Both (a) & (b) 16 Which of the following is/are false?. (a) India currently depends on images from a Canadian satellite as domestic remote sensing spacecraft cannot take pictures of the ground during cloud cover. (b) RISAT-1 would be particularly useful in Kharif season when cloud-covered atmosphere is frequent. (c) Neither (a) Nor (b) (d) Both (a) & (b) 17 Which of the following is/are false w.r.t. Review of the Economy 2011-12 presented by Dr. C. Rangarajan, Chairman, Economic Advisory Council to the Prime Minister?. (a) Farm sector output growth in 2011/12 has been strong. (b) While the electricity sector has performed well, manufacturing and construction have disappointed. (c) Neither (a) Nor (b) (d) Both (a) & (b) 18 Where is “The City of Puerto Princesa” which is among 'New 7 Wonders of Nature'? (a) Denmark (b)Venezuela (c) Philippines (d) Tanzania 19 Consider the following statements w.r.t. National Girl Child Day. 1. National Girl Child Day is celebrated on 24 January every year since 2010. 2. 24 January was chosen because on this date in 2010 that UN Women Forum was formed. (a) Only 1 is false (b) Only 2 is false (c) Both 1 and 2 are false (d) Neither 1 nor 2 is false 20 Indian Space Research Organisation has signed a deal, believed to be valued at around Rs 100 crore, to launch an 800 kg satellite of ASTRIUM. ASTRIUM is a/an (a) Aerospace subsidiary of the European Aeronautic Defence and Space Company (EADS). (b) Wing of European Space Agency. (c) Aerospace subsidiary of the Israeli Space Agency (d) Wing of Russian Federal Space Agency 21 Asia Cup Cricket Trophy 2012 was won by (a) India (b) Srilanka (c) Pakistan (d) Bangladesh 22 What are Pygmy marmosets? (a) They are the world's smallest monkey. (b) They are the closest relatives of bird in the evolutionary tree. (c) They were declared extinct recently by IUCN. (d) None of these. 23 Where is Khadakpurna Irrigation Project? (a) Chattisgarh (b) Madhya Pradesh
  • 3. Cracking IAS Study Circle, Chennai. PRELIMS 2012 SERIES Current Affairs - Questions _______________________________________________________ Address: - Y-block. 3020/1. 3rd street. 13th main road. Anna Nagar. Chennai-40 Landmark: Behind Natesan Institute of Co-operative Management on SHANTI COLONY ROAD. Call: us: 09884554654. www.crackingIAS.com (c) Uttarkhand (d) Maharashtra 24 A Titanium sponge plant, first of its kind in India is @ Chavara near Kollam in Kerala. Which of the following is/are true in this context? (a) It utilizes the technology developed by Defence Metallurgical Research Laboratory (DMRL), Hyderabad a laboratory under DRDO (Defence Research and Development Organization). (b) Vikram Sarabhai Space Centre, Trivandrum, which is a major user of titanium sponge, has funded the plant. (c) Both (a) & (b). (d) Neither (a) nor (b). 25 India's first aerotropolis or airport city project is at (a) West Bengal (b) Chennai (c) Delhi (d) Hyderabad 26 The first non-OECD, non-Council of Europe country to become a party to Multilateral Convention on Mutual Administrative Assistance in Tax Matters is (a) India (b) Pakistan (c) Bangladesh (d) Srilanka 27 Which state legislature in USA has passed a bill prohibiting companies doing business with Iran's energy industry from entering into contracts with the state (i.e)'Iran Divestment Act' in January 2012?. (a) New Jersey (b) New York (c) Virginia (d) Florida 28 Who is Jordan Romero who was in news recently? (a) Mountain Climber (b) Bio-Chemist (c) Aerospace specialist (d) Economist 29 Match the following List 1 List 2 A. Satinder Singh 1. Malaysia Bajaj B. Sultan Abdul Halim 2. Writer C. Jamil Ahmed 3. Atomic Energy 4. Jordan Codes A B C (a) 3 1 2 (b) 2 1 3 (c) 3 1 4 (d) 2 1 4 30 What is the objective of NASA’s “SPADE SYSTEM”? (a) To deorbit Space-Junk (b) To research on Saturn’s Ring system. (c) To research on Solar storms. (d) To establish an House in Mars for Human living purpose 31 Scientists have for the first time captured images of auroras above the giant ice planet Uranus, finding further evidence of just how peculiar a world that distant planet is. Identify the correct statement(s). (a) Unlike auroras on Earth, which can turn the sky greens and purples for hours, the newly detected auroras on Uranus appeared to only last a couple minutes. (b) These were detected by means of observations from the Hubble Space Telescope. (c) Neither (a) Nor (b) (d) Both (a) & (b) 32 A3P scheme is a part of (a) Gramin Bhandaran Yojana(GBY) (b) Rashtriya Krishi Vikas Yojana (c) Macro Management of Agriculture Scheme (d) National Food Security Mission 33 Which of the following countries has been requesting for Ghasuapara and Dalu in Meghalaya on Indo-Bangladesh border as on exit/entry points for its trade with Bangladesh?. (a) Bhutan (b) China (c) Nepal (d) Myanmar 34 The Indian Coast Guard Ship Rani Abbakka, the first of a series of five Inshore Patrol
  • 4. Cracking IAS Study Circle, Chennai. PRELIMS 2012 SERIES Current Affairs - Questions _______________________________________________________ Address: - Y-block. 3020/1. 3rd street. 13th main road. Anna Nagar. Chennai-40 Landmark: Behind Natesan Institute of Co-operative Management on SHANTI COLONY ROAD. Call: us: 09884554654. www.crackingIAS.com Vessel (IPV) built at Hindustan Shipyard Limited (HSL), was commissioned at Visakhapatnam. Which of the following is/are true about Rani Abbakka?. (a) She fought the French. (b) She fought the Dutch. (c) She fought the British. (d) She fought the Mughals. 35 Consider the following statements. 1. Jawai Dam is known for Crocodile Sanctuary. 2. Jawai river is a tributary of Luni river. (a) Only 1 is true (b) Only 2 is true (c) Both 1 and 2 are true (d) Neither 1 nor 2 is true 36 Consider the following statements. 1. ‘Punjab Mail’ is the first train to achieve distinction of entering its 100th year on Indian Railways broad gauge network. 2. “Punjab Mail” Train was the theme of Indian Railways Republic Day Tableau in 2012. (a) Only 1 is false (b) Only 2 is false (c) Both 1 and 2 are false (d) Neither 1 nor 2 is false 37 Who is Ali Ukla Ursan? (a) A writer (b) An aerospace specialist (c) A nuclear scientist (d) An aide of Osama Bin Laden 38 When was the constitution amended to lower the voting age from 21 years to 18 years?. (a) 1988 (b) 1989 (c) 1987 (d) 1990 39 What is “Bheem” which was in news recently?. (a) A newly designed diesel locomotive. (b) A newly designed fuel efficient car. (c) A newly formulated energy enhancer for athletes by DRDO. (d) A newly designed Battle Tank made by HVF Avadi. 40 Which of the following countries along with Russia was/were acceded as members of WTO in December 2011? (1) Montenegro (2) Samoa (3) Vanuatu (a) 1, 2 & 3 (b) 1 & 2 (c) Only 1 (d) 1 & 3 41 The term “MESONET” is related to (a) Mobile Phone (b) Nuclear Energy (c) Genetics (d) Meteorology 42 Consider the following statements. 1. Pradhan Mantri Adarsh Gram Yojana (PMAGY) is under Ministry of Social Justice and Empowerment. 2. Pradhan Mantri Adarsh Gram Yojana (PMAGY) is for the integrated development of scheduled castes dominated villages (a) Only 1 is true (b) Only 2 is true (c) Both 1 and 2 are true (d) Neither 1 nor 2 is true 43 The only Indian bank in the Top 17 BRIC banks, as listed by Global Finance (a) ICICI (b) State Bank of India (c) Punjab National Bank (d) Indian Overseas Bank 44 Who among the following is Cuba’s tourism ambassador? (a) Abishek Bachan (b) Amitab Bachan (c) Sharukh Khan (d) Salman Khan 45 The Ministry of Health and Family Welfare announced the launch of Weekly Iron and Folic Supplementation (WIFS) programme which will target (a) Children (b) Adolescents (c) Pregnant women (d) Both (a) and (c) 46 An elaborate commentary on Aryabhatiya called the Aryabhatiya Bhasya was composed by (a) Nilakantha
  • 5. Cracking IAS Study Circle, Chennai. PRELIMS 2012 SERIES Current Affairs - Questions _______________________________________________________ Address: - Y-block. 3020/1. 3rd street. 13th main road. Anna Nagar. Chennai-40 Landmark: Behind Natesan Institute of Co-operative Management on SHANTI COLONY ROAD. Call: us: 09884554654. www.crackingIAS.com (b) Baudhayana (c) Brahmagupta (d) Varahamihira 47 Where is Yoloten-Osman gas field?. (a) Iran (b) Turkmenistan (c) Iraq (d) Jordan 48 Which of the following is/are true about Operation Cobra Gold 2012? (a) It is military exercise sponsored by USA & Thailand. (b) India is a full participating nation in this exercise. (c) Both (a) and (b) (d) Neither (a) nor (b) 49 Which of the following cities has earned the sobriquet of `Orange City’? (a) Salem (b) Nagpur (c) Pune (d) None of these. 50 Which of the following is/are false w.r.t India State of Forest Report 2011 released in February 2012? (a) The Forest and Tree cover is 23.81% of the geographical area of the country. (b) The state of Madhya Pradesh has the largest forest cover in the country followed by Arunachal Pradesh. (c) Neither (a) Nor (b) (d) Both (a) & (b) 51 “Lassa fever claims 13 lives in Nigeria” was the recent headline news. Which of the following is/are true in this context?. (a) Lassa is a virus. (b) It is transmitted primarily though contact with rat excreta. (c) Neither (a) Nor (b) (d) Both (a) & (b) 52 'Janitor satellite' made to clean up space junk – recent headline news. Which country has made this?. (a) Russia (b) USA (c) Switzerland (d) China 53 Who was Ramkinkar Baij? (a) Sculptor (b) Violinist (c) Environmental activist (d) IT sector entrepreneur 54 Which of the following is/are true about “Durban Platform for Enhanced Action, December 2011” in news recently?. (a) A new legally binding treaty to cut greenhouse gases, expected to be decided by 2015 and come into force by 2020. (b) A goal of keeping global temperature increases to “1.5 or 2.0 degrees C” above preindustrial levels. (c) Neither (a) Nor (b) (d) Both (a) & (b) 55 In what context 'Sejeal stickers' were in news recently? (a) A stamp released on the eve of launch of Project Seal to conserve seals (b) A newly discovered high power adhesive (c) An use and throw harmless tattoo (d) None of these 56 Asia's first dolphin research centre is to come up in (a) Bihar (b) West Bengal (c) Chennai (d) Goa 57 Sahyog-Kaijin is a joint exercise between (a) India, Srilanka Coast Guards (b) India, Japan Coast Guards (c) India, Indonesia Coast Guards (d) India, Australia Coast Guards 58 W.r.t. the Eighth Ministerial Conference of WTO, 2011 which of the following is/are true?. (a) India has strongly opposed a proposal by the EU and a few other developed countries to include new issues including energy & food security. (b) India has ruled out freezing of custom duties at the current levels. (c) India has ruled out any dilution of the flexibilities available under the WTO regime for imposing export restrictions and taxes. (d) 1, 2 & 3
  • 6. Cracking IAS Study Circle, Chennai. PRELIMS 2012 SERIES Current Affairs - Questions _______________________________________________________ Address: - Y-block. 3020/1. 3rd street. 13th main road. Anna Nagar. Chennai-40 Landmark: Behind Natesan Institute of Co-operative Management on SHANTI COLONY ROAD. Call: us: 09884554654. www.crackingIAS.com 59 First player to take 500 catches in Test cricket (a) Mark Boucher (b) Phillip Hughes (c) Vernon Philander (d) Sachin Tendulkar 60 Which of the following state has the lowest and highest Infant Mortality respectively?. (a) Goa & Madhya Pradesh (b) Goa & Bihar (c) Kerala & Orissa (d) Kerala & Bihar 61 Which of the following Baltic country/(ies) recently rejected Russian as official language?. (a) Estonia (b) Latvia (c) Lithuania (d) Both (a) and (b) 62 As of December 2011 New Parliamentary Committee on welfare of Other Backward Classes is to be constituted. Which of the following is/are true w.r.t this? (a) The Committee will comprise 30 members, 20 from Lok Sabha and 10 from Rajya Sabha. (b) The members of the Committee shall hold office for a period of one year from the date of the first meeting and the quorum to constitute a sitting shall be 10. (c) Neither (a) Nor (b) (d) Both (a) & (b) 63 What is “Stevia” which was in news recently? (a) An indigenously made hydrogen fuelled car. (b) A clone of Jersey cow recently made by NDRI. (c) A Cyclone which hit Philippines recently. (d) None of these. 64 As of December 2011 how many of the states are yet to constitute state HRCs? (a) 6 (b) 8 (c) None (d) Only One 65 Scientists have come up with a new wireless device which they claim can detect the presence of termites. Which of the following is/are true in this regard?. (a) It does so by "hearing" them chew through timber. (b) The device can communicate the termites' GPS location. (c) Neither (a) Nor (b) (d) Both (a) & (b) 66 Which of the following is/are the components of the Jawaharlal Nehru National Urban Renewal Mission (JNNURM)?. (1) Basic Services to the Urban Poor (2) Urban Infrastructure and Governance (3) Peer Experience and Reflective Learning (a) Only 1 (b) Only 1 & 2 (c) 1, 2 & 3 (d) Only 1 & 3 67 “World's worst soccer violence in 15 years”. This was the recent headline news. Where did this happen?. (a) Egypt (b) Brazil (c) Argentina (d) Syria 68 The first ever vaccine to prevent kala azar, or visceral leishmaniasis (VL), the world's second largest parasitic killer after malaria, is entering dual clinical trials in the (a) US & Japan (b) US & India (c) India & Canada (d) Japan & Canada 69 Global Wind Energy Council’s (GWEC) annual statistics (2011) shows (a) India had the most cumulative installed wind power capacity. (b) In second place was China in terms of cumulative installed wind power capacity. (c) Neither (a) Nor (b) (d) Both (a) & (b) 70 “Scientists capture footage of rare whale” was the recent headline news. Which of the following is/are true in this context?. (a) This was accomplished by The Australian Antarctic Division team. (b) These whales have dolphin-like beaks. (c) Neither (a) Nor (b) (d) Both (a) & (b)
  • 7. Cracking IAS Study Circle, Chennai. PRELIMS 2012 SERIES Current Affairs - Questions _______________________________________________________ Address: - Y-block. 3020/1. 3rd street. 13th main road. Anna Nagar. Chennai-40 Landmark: Behind Natesan Institute of Co-operative Management on SHANTI COLONY ROAD. Call: us: 09884554654. www.crackingIAS.com 71 How many destinations have been identified by the Union Ministry of Tourism to be developed as Mega Tourist Hubs across India (as of January 2012)? (a) 45 (b) 30 (c) 35 (d) 40 72 Bureau of Indian Standards is under (a) Ministry of Consumer Affairs (b) Ministry of Science & Technology (c) Ministry of Commerce & Industry (d) None of these 73 The Union budget has announced a provision for the National Electricity Fund (NEF). National Electricity Fund provides (a) Interest subsidy for projects of electricity distribution sector. (b) Interest subsidy for projects of electricity production sector. (c) Interest subsidy for projects of solar energy production sector. (d) Interest subsidy for projects of eco friendly energy production sector. 74 The figure here denotes a songbird that weighs a mere 25 grams and completes the longest known flight over water of any songbird — 3,500 kilometres across the Atlantic. It spends the winter in Africa. Which are their breeding grounds? (a) Canadian Arctic (b) Alaska (c) Neither (a) Nor (b) (d) Both (a) & (b) 75 Who heads Loomba Foundation which is working for the welfare of widows?. (a) Lakshmi Mittal (b) Bill Gates (c) Cherie Blair (d) Indra Nooyi CSAT (Paper 1 & 2) 2012 FINAL MOCK TEST @ Chennai on April 29, 2012 PAPER 1 Afternoon 2.30 p.m. - 4.30 p.m. PAPER 2 April 30, 2012 Afternoon 2.30 p.m. - 4.30 p.m. Prior Registration is Must. Fees – Only Rs.50. Call:SMS:9884554654
  • 8. Cracking IAS Study Circle, Chennai. PRELIMS 2012 SERIES Current Affairs - Questions _______________________________________________________ Address: - Y-block. 3020/1. 3rd street. 13th main road. Anna Nagar. Chennai-40 Landmark: Behind Natesan Institute of Co-operative Management on SHANTI COLONY ROAD. Call: us: 09884554654. www.crackingIAS.com @ CHENNAI - Classroom Guidance (Next Batch Classes for IAS 2013 & Mains 2012) POLITICAL SCIENCE & GS by Raja Sir & Team. You cannot get a better guidance in Chennai. Our Old students certify us. I made a brilliant choice by joining Cracking IAS Classes @ Chennai. Arihant. Hyderabad. (In an interview to TV Channel) Mains 2012 Test Series @ Chennai Highly probable questions for MAINS 2012 Address Cracking IAS, Y-block. 3020/1. 3rd street. 13th main road. Anna Nagar. Chennai-40 Landmark: Behind Natesan Institute of Co-operative Management on Shanti Colony Road 500+ marks in GS Mains 2011 - Few examples (GS Mains 2010) Paper 1 Q. 7(a) in – Our Mains current affairs notes P.no.176. Q. 8(c) in Paper 1 – Our Mains current affairs notes P.no.200. Q. 9(f) & (g) in Paper 1 – Our Model test questions...& many more… Paper 2: Q. 2(a) & (c) – Our India and World notes and also our model test question. Q. 3(a) & (d) – Our India and World notes. Q. 3(f) – Our Science and Technology notes. Q. 3(h) – Our Mains current affairs notes P.no.136. Q. 5(b) – Our IAS Express August 2010 P.no.47. Q. 5(d) – Our Science and Technology notes and also our model test question.. Q. 7(a) – Our Mains current affairs Science and Technology notes P.no.1... & many more…Due to space constraints we could not list the other questions… That’s because We sweat 4 ur SWEET SUCCESS! Excel with our Expertise in SOCIOLOGY Many of our students scored 320+ in Sociology in Mains 2010 with our member’s highest score being 357 in Mains 2010. SOCIOLOGY by Raja Sir Conquer the corridors of power with our Power Packed Political Science classes @ Chennai. Our member scored 340 in Political Science Mains. 190 in Political Science Paper 2 - AIR – 2 as per our known records (IAS ‘07). MAINS 2012 SURE SHOT Special GS course @ Chennai, From June 1st week, 2012 An early bird Discount of 10% for those get admission before May 10th 2012. CSAT By toppers in Aptitude Essay Enrichment Programme @ Chennai from August, 2012 Call: us: 09884554554554554654
  • 9. Cracking IAS Study Circle, Chennai. PRELIMS 2012 SERIES Current Affairs - Questions _______________________________________________________ Address: - Y-block. 3020/1. 3rd street. 13th main road. Anna Nagar. Chennai-40 Landmark: Behind Natesan Institute of Co-operative Management on SHANTI COLONY ROAD. Call: us: 09884554654. www.crackingIAS.com 1 Under the concept of the Spice Park, Board will lease out the lands available in the Spice Park to private entrepreneurs for developing their own processing plants for value addition. 2nd Spices park in India was established @ Jodhpur, Rajasthan. A 2 • Headed by C. Rangarajan, Chairman of the Economic Advisory Council to the Prime Minister, the Committee has been requested to complete its task as early as possible and give its recommendations to the Prime Minister. • The sugar industry has been demanding de-control of the sector on the plea that the mandatory requirement of selling 10 per cent of production at below-cost rates for supply to the ration shops is crippling it. • Another key demand is to do away with the system under which the Food Ministry decides on the quantity of the sugar that can be sold in the open market every month. C 3 The Indian Bureau of Mines (IBM) is a subordinate office under the Ministry of Mines. It is engaged in the promotion of scientific development of mineral resources of the country, conservation of minerals, protection of environment in mines for minerals, other than coal, petroleum and natural gas, atomic mineral and minor minerals. It performs regulatory functions, namely enforcement of the Mineral Conservation and Development Rules, 1988, the relevant provisions of the Mines and Minerals (Development and Regulation) Act, 1957, Mineral Concession Rules, 1960 and Environmental (Protection) Act, 1986 and Rules made there under. It also undertakes scientific, techno-economic, research oriented studies in various aspects of mining, geological studies, ore beneficiation and environmental studies. C 4 The State of Karnataka has set up a tiger protection force of fifty four foresters and forest guards trained to protect the tigers its reserves. This is a first step of its kind ever taken in India. The STPF has been constituted under the directive of MOEF this unit will be initially in charge of Billgiri Rangaga Temple Wildlife Sanctuary, Bandipur Tiger Reserve and Nagarhole Tiger Reserve in the state. The major threat to the tigers in Southern States is from the infamous Khatni Gang active since last twenty five years in the country. A 5 Prime Minister Manmohan Singh recently held talks with his Mauritian counterpart Navinchandra Ramgoolam on the issue even as the two countries vowed to strengthen security cooperation and to jointly battle the menace of piracy. B 6 C
  • 10. Cracking IAS Study Circle, Chennai. PRELIMS 2012 SERIES Current Affairs - Questions _______________________________________________________ Address: - Y-block. 3020/1. 3rd street. 13th main road. Anna Nagar. Chennai-40 Landmark: Behind Natesan Institute of Co-operative Management on SHANTI COLONY ROAD. Call: us: 09884554654. www.crackingIAS.com 7 It is a Belgian scientific polar research station. A 8 D 9 Two Indian-origin men, including a journalist, are among South Africa's 10 heroes who contributed to the public life in 2011. Journalist Yusuf Abramjee and philanthropist Dr Imtiaz Sooliman joined the list, compiled by a newspaper, which was topped by South African Public Protector Thuli Madonsela. D 10 C 11 The completed Golden Quadrilateral will pass through 13 States of India:- • Andhra Pradesh – 1,014 km, Uttar Pradesh, Rajasthan, Karnataka, Maharashtra, Gujarat, Orissa, West Bengal, Tamil Nadu, Bihar, Jharkhand, Haryana, Delhi – 25 km. The GQ project is managed by the National Highways Authority of India (NHAI) under the Ministry of Road, Transport and Highways. • The Mumbai-Pune Expressway, the first controlled-access toll road to be built in India is a part of the GQ Project though not funded by NHAI, and separate from the main highway. Infrastructure Leasing & Financial Services (IL&FS) has been one of the major contributors to the infrastructural development activity in the GQ project. C 12 C 13 India’s largest iron ore producer and exporter, NMDC Ltd, has announced the conclusion of acquisition of a 50 per cent equity stake in Perth-based Australian iron ore, gold and base metals exploration company Legacy Iron Ore Ltd for a consideration of A$18.89 million. According to a statement made to the stock exchanges, Legacy based in Western Australia, is developing the Mt Bevan Iron Ore project, which has the potential to be one of the largest resources of iron ore in Central Yilgarn region. The company is also developing an advanced gold exploration project at Mt. Celia and holds other prospective areas for iron and manganese and also gold at East Kimberley. C 14 D 15 C 16 C 17 C 18 The Philippines' Puerto Princesa Underground River, the Amazon rainforest, Vietnam's Halong Bay and Argentina's Iguazu Falls were named among the world's new seven wonders of nature, according to organizers of a global poll. The other three crowned the C
  • 11. Cracking IAS Study Circle, Chennai. PRELIMS 2012 SERIES Current Affairs - Questions _______________________________________________________ Address: - Y-block. 3020/1. 3rd street. 13th main road. Anna Nagar. Chennai-40 Landmark: Behind Natesan Institute of Co-operative Management on SHANTI COLONY ROAD. Call: us: 09884554654. www.crackingIAS.com world's natural wonders are South Korea's Jeju Island, Indonesia's Komodo, and South Africa's Table Mountain, said the New7Wonders foundation, citing provisional results. 19 National Girl Child Day is celebrated on 24 January every year since 2009. It was on this date in 1966 Smt Indira Gandhi became the first woman Prime Minister of India. On December 19, 2011, a historic milestone for girls' rights was achieved when the UN General Assembly formally declared October 11 as the International Day of the Girl Child. B 20 Under this agreement, an advanced Remote Sensing satellite - SPOT-6 - weighing nearly 800 kg, built by ASTRIUM SAS, will be launched on-board ISRO's Polar Satellite Launch Vehicle, during the second half of 2012. In November 2010, under a commercial contract between ANTRIX and ASTRIUM, an advanced communication satellite HYLAS was successfully built by ISRO and ASTRIUM together for a European customer. A 21 Pakistan defeated host Bangladesh to lift Asia Cup Cricket Trophy on 22 March 2012. C 22 Two pygmy marmosets, known as the world's smallest monkey species, have been brought to Sakkarbaug Zoo in Junagadh under an exchange programme from Mysore Zoo. A pigmy marmoset is just 5.5 to 6.5 inches 'tall', and weighs 300 to 500 grams. The species is found in Western Brazil, South-Eastern Colombia, Eastern Ecuador and Eastern Peru, zoo officials said, adding that the tiny animal's diet includes fruit, leaves, insects, small reptiles and tree-sap. A 23 D 24 • India has joined an exclusive club with the knowhow to process titanium sponge, a key material in space and defence applications. ISRO has always depended on overseas markets to source titanium sponge, which is created during the first stage of processing titanium ore. It is the raw material used to make alloys that are used to make rockets. The job of converting titanium sponge to titanium alloy will be done by the Hyderabad-based Mishra Dhatu Nigam. • Titanium ore is mined from beaches and the metal is known for its high strength but low weight, making it an ideal material for aircraft manufacture, including fighter aircraft. The material is also used in nuclear plants, and to make dental implants and mobile phones. • China dominates the production and use of titanium. Nine out of 18 companies C
  • 12. Cracking IAS Study Circle, Chennai. PRELIMS 2012 SERIES Current Affairs - Questions _______________________________________________________ Address: - Y-block. 3020/1. 3rd street. 13th main road. Anna Nagar. Chennai-40 Landmark: Behind Natesan Institute of Co-operative Management on SHANTI COLONY ROAD. Call: us: 09884554654. www.crackingIAS.com making titanium sponge are Chinese. The KMML plant not only helps India get parity in strategic affairs but also opens the door for regional influence. In the pipeline is a partnership to make titanium sponge with Kazakhstan. This is another opportunity for India to expand its sphere of influence as a provider of special materials. • Other research programmes undertaken by the KMML is also progressing. This includes the novel “nano titanium pigment” which has tremendous scope in the market. Its market price is ten times the price of titanium pigment, one of the current products of KMML. The waste generated at the mineral separation unit of the KMML is also being put to use. • Titanium sponge is a porous form of titanium that is created during the first stage of processing. In its natural form, titanium is widely available within the earth’s crust. After being extracted, it is processed to remove excess materials and convert it into a usable, although costly, product. The conversion process used with raw titanium is called the KROLL PROCESS. 25 Country's first aerotropolis or airport city project at Andal in West Bengal will be operational in the last quarter of 2012. A 26 India had signed the Multilateral Convention on Mutual Administrative Assistance in Tax Matters on 26th January, 2012. The Convention has been ratified by India by depositing the Instrument of Ratification as on 21st February, 2012. By this, India has become the first non-OECD, non-Council of Europe country to become a party to the Convention as amended by the 2010 Protocol A 27 It would prohibit companies that provide goods, services or credit worth USD 20 million or more to Iran's energy industry from entering into or renewing state and local government contracts. B 28 An American teenager has become the youngest person to climb to the summit of the highest mountains on each of the seven continents. Jordan Romero, 15, reached the 4,897m (16,067ft) summit of Vinson Massif in Antarctica on 24.12.2011, the final peak in a quest he began six years ago. 'Seven summits' Africa - Mt Kilimanjaro (5,892m; 19,340ft) Antarctica - Vinson Massif (4,892; 16,050) Australia - Mt Kosciuszko (2,228m; 7,310ft) A
  • 13. Cracking IAS Study Circle, Chennai. PRELIMS 2012 SERIES Current Affairs - Questions _______________________________________________________ Address: - Y-block. 3020/1. 3rd street. 13th main road. Anna Nagar. Chennai-40 Landmark: Behind Natesan Institute of Co-operative Management on SHANTI COLONY ROAD. Call: us: 09884554654. www.crackingIAS.com Oceania - Carstensz Pyramid (4,884m; 16,024ft) Asia - Mt Everest (8,848m; 29,035ft) Europe - Mt Elbrus (5,642m; 18,510ft) North America - Mt McKinley (6,194m; 20,320ft) South America - Aconcagua (6,962m; 22,841ft) Highest mountains of each of the seven continents. 29 A 30 A 31 Auroras are a feature of the magnetosphere, the area surrounding a planet that is controlled by its magnetic field and shaped by the solar wind, a steady flow of charged particles emanating from the sun. Auroras are produced in the atmosphere as charged solar wind particles accelerate in the magnetosphere and are guided by the magnetic field close to the magnetic poles – that's why the Earthly auroras are found around high latitudes. Auroras on Uranus are fainter than they are on Earth, and the planet is more than 4 billion kilometers (2.5 billion miles) away. Previous Earth-bound attempts to detect the faint auroras were inconclusive. Astronomers got their last good look at Uranian auroras 25 years ago when the Voyager 2 spacecraft whizzed past the planet and recorded spectra from of the radiant display. D 32 Accelerated pulses production programme (A3P) has been launched under the aegis of National Food Security Mission (NFSM Pulses ), a Centrally-sponsored scheme. D 33 A 34 She fought the Portuguese. She was also one of the earliest Indians to fight the colonial powers and is sometimes regarded as the 'first woman freedom fighter of India'. Her story is a legend in Karnataka, and is often enacted in Yakshagana, a popular folk theatre style in Tulu Nadu. A 35 Keshopur Chhamb Community Reserve in Gurdaspur District of Punjab, declared in 2007, is the first Community Reserve in the country. Recently, Rajasthan government in India declared "Jawai Bandh forests" as a conservation reserve forest. Jawai Bandh forest is situated in Pali district and it is in close proximity of Kumbalgarh Sanctuary. (under Section 36 of the amended Wildlife Protection Act, 1972). Located in the south-western transition zone of the Aravallis on the Jawai river and the Luni river basin, and in close proximity of Kumbalgarh sanctuary, the Jawai dam was built in 1957 for drinking water purposes. Considered to be an ideal location for sighting crocodiles, the area also has a C
  • 14. Cracking IAS Study Circle, Chennai. PRELIMS 2012 SERIES Current Affairs - Questions _______________________________________________________ Address: - Y-block. 3020/1. 3rd street. 13th main road. Anna Nagar. Chennai-40 Landmark: Behind Natesan Institute of Co-operative Management on SHANTI COLONY ROAD. Call: us: 09884554654. www.crackingIAS.com large presence of turtles, fishes and aquatic birds. The Jawai Bandh terrain was declared a closed area in 1983 and after the 2002 amendments it ceased to exist in that category. 36 D 37 Amid the escalating violence in Syria and a diplomatic outcry over Russia’s actions blocking intervention by the United Nations, President Dmitri A. Medvedev bestowed a prestigious cultural award on a Syrian writer Ali Ukla Ursan and poet who has publicly applauded the terrorist attacks of Sept. 11, 2001, and expressed strong anti-Semitic views. A 38 The 61st Amendment act 1988 reduced the voting age from 21 years to 18 years for the Lok Sabha and Assembly election. The Election commission has declared 25th January, its foundation day, as National Voters Day. C 39 With an aim of hauling heavier trains, railways is ready to roll out a newly-designed 5500 horse power (HP) diesel locomotive equipped with all modern features including 100 km per hour running speed. Aptly named as “Bheem", the high horse power locomotive is currently undergoing safety trial. Manufactured at Diesel Locomotive Works at Varanasi, the prototype has cost about Rs 17 crore to the railways. Designed jointly by the Railways and EMD, a US-based company, the locomotive has a provision of toilet for crew, a first in railways. A 40 Russia acceded as a member, along with Samoa, Montenegro and Vanuatu. D 41 In meteorology, a mesonet is a network of automated weather stations designed to observe mesoscale meteorological phenomena. The forecasting of the weather within the next six hours is often referred to as nowcasting. D 42 The PMAGY was announced in 2009-10 for integrated development of all villages with more than 50 percent scheduled caste population. Presently, there are 44,000 such villages. As a pilot project, it has been implemented in 1,000 villages in Assam, Bihar, Himachal Pradesh, Rajasthan and Tamil Nadu with an allocation of Rs. 100 crore with each village to get Rs. 10 lakh per year. The Planning Commission has cleared the transfer of the Pradhan Mantri Adarsh Gram Yojana (PMAGY) to the Ministry of Rural Development (MoRD) from the Ministry of Social Justice and Empowerment. (MARCH 2012). b 43 B 44 D
  • 15. Cracking IAS Study Circle, Chennai. PRELIMS 2012 SERIES Current Affairs - Questions _______________________________________________________ Address: - Y-block. 3020/1. 3rd street. 13th main road. Anna Nagar. Chennai-40 Landmark: Behind Natesan Institute of Co-operative Management on SHANTI COLONY ROAD. Call: us: 09884554654. www.crackingIAS.com 45 • The Ministry has suggested to the States that a fixed day in a week, preferably a Monday, be earmarked as the day when Iron and Folic Acid tablet is provided to adolescents. • Funding for implementation of the scheme to the States would be provided under the National Rural Health Mission. • The States have been advised to project their fund requirements in the Programme Implementation Plans for the year 2012-13 so that the scheme can be rolled out in the forthcoming financial year. • The key features of WIFS include administration of supervised Weekly Iron-folic Acid Supplements of 100mg elemental iron and 500ug Folic acid; screening of target groups for moderate/severe anaemia and referring these cases to an appropriate health facility; Information and counselling for improving dietary intake and for taking actions for prevention of intestinal worm infestation. B 46 In this Bhasya, Nilakantha had discussed infinite series expansions of trigonometric functions and problems of algebra and spherical geometry. Grahapareeksakrama is a manual on making observations in astronomy based on instruments of the time. A 47 The 1,735-km-long TAPI pipeline will run from Turkmenistan's Yoloten-Osman gas field to Herat and Kandahar province of Afghanistan, before entering Pakistan. In Pakistan, it will reach Multan via Quetta before ending at Fazilka (Punjab) in India. India and Pakistan recently agreed in-principle to have a uniform transit fee for ferrying natural gas through the proposed USD 7.6 billion pipeline from Turkmenistan. India will pay a transit fee to Pakistan and Afghanistan for getting its share of 38 million standard cubic metres per day of gas through the Turkmenistan-Afghanistan-Pakistan-India pipeline, while Islamabad has to pay ferrying charges only to Afghanistan for allowing passage of the fuel. B 48 'Exercise Cobra Gold 2012' is designed to advance regional security by exercising a robust multi-national force from nations sharing common goals and security commitments in the Asia-Pacific region. Full participating nations for Cobra Gold 12 include Thailand, the US, Singapore, Japan, South Korea, Indonesia and Malaysia. Several Countries which have been invited to participate on the multi-national planning augmentation team included Australia, France, Canada, the United Kingdom, Bangladesh, A
  • 16. Cracking IAS Study Circle, Chennai. PRELIMS 2012 SERIES Current Affairs - Questions _______________________________________________________ Address: - Y-block. 3020/1. 3rd street. 13th main road. Anna Nagar. Chennai-40 Landmark: Behind Natesan Institute of Co-operative Management on SHANTI COLONY ROAD. Call: us: 09884554654. www.crackingIAS.com Italy, India, Nepal, Philippines and Vietnam 49 Nagpur lies precisely at the center of the country with the Zero Mile Marker indicating the geographical center of India. Nagpur is also declared, "Tiger Capital of India" as it connects many Tiger Reserves in India to the world. B 50 In terms of percentage of forest cover in relation to total geographical area, Mizoram tops with 90.68% followed by Lakshadweep with 84.56%. The India State of Forest Report 2011 contains the regular features like forest cover, tree cover, mangroves and growing stock both in forests and areas outside forests. However, it adds three new chapters that are of crucial importance in the present national and global worldview about forests. These are: a detailed assessment of bamboo resources, a production-consumption assessment of wood based on data stock in India’s forests reported under the NATCOM project. C 51 Lassa fever is an acute viral hemorrhagic fever caused by the Lassa virus and first described in 1969 in the town of Lassa, in Borno State, Nigeria, in the Yedseram river valley at the south end of Lake Chad. The primary animal host of the Lassa virus is the Natal Multimammate Mouse (Mastomys natalensis), an animal indigenous to most of Sub- Saharan Africa. The virus is probably transmitted by contact with the feces or urine of animals accessing grain stores in residences. D 52 C 53 Ramkinkar Baij - the founding father of modern Indian sculpture. A 54 D 55 These stickers were found in the Bangkok house rented by the bungling Iranian bombers has left Thai police puzzled. The word 'sejeal' is a reference to a miracle related in the Quran, wherein birds pelted the enemies of the Prophet with baked clay 'stones' called sejeals. Palestinians refer to their rockets and mortar shells as sejeal stones, and the Islamic Republic of Iran makes a missle called 'sijjil'. Police in Thailand say three Iranian nationals held in connection with a bomb blast in the Thai capital were plotting to attack Israeli diplomats. D 56 The man behind the proposal is R.K. Sinha, an expert on Gangetic river dolphins and chairperson of the working group for dolphin conservation set up by the central government. He said the centre was suggested by the Planning Commission and subsequently received "in principle" approval by the state government. The Vikramshila Gangetic Dolphin Sanctuary, India's only dolphin sanctuary, spread over 50 km along the Ganges, is located in Bihar's Bhagalpur district. The Gangetic river dolphin is India's A
  • 17. Cracking IAS Study Circle, Chennai. PRELIMS 2012 SERIES Current Affairs - Questions _______________________________________________________ Address: - Y-block. 3020/1. 3rd street. 13th main road. Anna Nagar. Chennai-40 Landmark: Behind Natesan Institute of Co-operative Management on SHANTI COLONY ROAD. Call: us: 09884554654. www.crackingIAS.com national aquatic animal but frequently falls prey to poachers. Their carcasses are found regularly on river banks. Gangetic river dolphins fall under Schedule I of the Indian Wildlife (Protection) Act and have been declared an endangered species by the International Union for Conservation of Nature (IUCN). The Gangetic river dolphin is one of the four freshwater dolphin species in the world. The other three are found in the Yangtze river in China, the Indus river in Pakistan and the Amazon river in South America. The Gangetic river species - found in India, Bangladesh and Nepal - is blind and finds its way and prey in the river waters through 'echoes'. 57 B 58 G33 countries is a coalition of agricultural economies with defensive interests in agriculture, coordinated by Indonesia. D 59 Mark Boucher, the South African wicketkeeper, has become the first cricketer to take 500 Test catches during the opening session of the first Test against Australia on Wednesday. His 500th dismissal was that of Australian opener Phillip Hughes who was caught behind off the bowling of Test debutant Vernon Philander. A 60 Goa has the lowest IMR of 10 infant deaths followed by Kerala with 13 infant deaths per 1,000 live births. Madhya Pradesh has the highest IMR of 62 infant deaths per 1,000, followed by Uttar Pradesh and Odisha. A 61 The term Baltic states - Estonia, Latvia, Lithuania (from north to south). B 62 The Committee will consider the reports submitted by the National Commission for Backward Classes and report to both the Houses on the measures that should be taken by the Union government within its purview and the Administrations of the Union Territories. D 63 Stevia leaves contain stevioside, which is 300 times sweeter than sugar. Artificial sweeteners have been mired in controversy as some of them have been found to be carcinogenic. On July 26, Food Safety and Standards Authority of India (FSSAI) issued a notification clarifying that stevia is not a food item. However, currently a scientific panel in FSSAI is considering whether stevia can be approved as a food item. A 64 States which have not constituted the state human rights commissions are Arunachal Pradesh, Goa, Haryana, Meghalaya, Mizoram, Nagaland, Tripura and Uttarakhand. B 65 A team at Edith Cowan University says that the sensitive device, called wireless smart D
  • 18. Cracking IAS Study Circle, Chennai. PRELIMS 2012 SERIES Current Affairs - Questions _______________________________________________________ Address: - Y-block. 3020/1. 3rd street. 13th main road. Anna Nagar. Chennai-40 Landmark: Behind Natesan Institute of Co-operative Management on SHANTI COLONY ROAD. Call: us: 09884554654. www.crackingIAS.com probe (WiSPr) network for acoustic detection, is also capable of locating termite activity in timber bridges and wooden power poles. 66 "Peer Experience and Reflective Learning" (PEARL) is an initiative under JNNURM to support cities to actively pursue activities in implementation of projects and reforms. C 67 A 68 The trials funded by the Bill & Melinda Gates Foundation are launched by the Infectious Disease Research Institute (IDRI), a Seattle-based non-profit research body with a companion Phase 1 trial planned in India, an epicenter of the disease. IDRI is transferring its technology to Pune's Gennova Biopharmaceuticals, which has opened a new facility that will produce the candidate vaccine for the clinical trial to begin late in 2012. The phase-1 trial in the US has been launched in Washington state in the US. The IDRI vaccine, known as LEISH-F3 + GLA-SE, is a highly purified, recombinant vaccine. It incorporates two fused Leishmania parasite proteins and a powerful adjuvant to stimulate an immune response against the parasite. B 69 China had the most cumulative installed wind power capacity — 62,733 MW — at the end of 2011, followed by the United States with 46,919 MW. Germany was in third place with 29,060 MW followed by Spain with 21,674 MW. In fifth place was India with 16,084 MW. C 70 D 71 For each identified mega destination and circuit, the Ministry of Tourism contribution as central financial assistance is capped at Rs.25.00 crore and Rs.50.00 crore respectively under the scheme of “Product/Infrastructure Development for Destinations and Circuits”. A 72 A 73 The fund is being set up to provide interest subsidy on loans to be disbursed to the Distribution Companies (DISCOMS) - both in the public and private sector, to improve the distribution network for areas not covered by Rajiv Gandhi Gramin Vidyutikaran Yojana (RGGVY) and Restructured Accelerated Power Development and Reforms Programme (R-APDRP) project areas. For the first time, India is planning a mandatory rating system for 65 state-owned distribution firms. Once the government assigns ratings to SEBs, they will form the basis on which state-controlled banks and financial institutions will lend to them. If a utility is highly rated, it will be eligible for more funds at a lower rate of interest. The Centre has set up the fund following recommendation of the V K Shunglu panel. A
  • 19. Cracking IAS Study Circle, Chennai. PRELIMS 2012 SERIES Current Affairs - Questions _______________________________________________________ Address: - Y-block. 3020/1. 3rd street. 13th main road. Anna Nagar. Chennai-40 Landmark: Behind Natesan Institute of Co-operative Management on SHANTI COLONY ROAD. Call: us: 09884554654. www.crackingIAS.com The panel recommended that the states be asked to take over distressed loans of discoms if they want to avail financial assistance from the Centre. Besides, they should also commit on implementation of power reforms in a time bound manner while negotiating terms of availing central financial package. 74 For the first time, Ontario scientists have proven what they long suspected — the wheatear is the only songbird that breeds in the Canadian Arctic and Alaska and spends the winter in Africa. The migration can take from one to three months. They also showed that the wheatear completes the longest known flight over water of any songbird — 3,500 kilometres across the Atlantic. That remarkable passage takes about four days of non-stop flying. There are two separate populations of wheatears in the far north. One breeds in Alaska, flies west over Russia, Kazakhstan and the Arabian Desert some 14,500 km, ending up in Sudan and Uganda in East Africa. Another, which breeds in the Canadian Arctic near Iqaluit heads the opposite direction, east over the Atlantic, stopping for the winter in Mauritania in West Africa, a journey of 7,500 km. The species is of particular interest to scientists, because it has one of the largest ranges of any songbird in the world. D 75 In 2005, Loomba Foundation president Cherie Blair launched International Widows Day at the House of Lords in London and over the next five years, the Foundation campaigned for international recognition of this day as a focus for sustained, effective, global action to bring about a radical and lasting transformation in the plight of widows. The significance of 23 June is that this is the day, in 1954, that the woman who inspired the founding of the Loomba Foundation, Shrimati Pushpa Wati Loomba, became a widow. The international goodwill ambassador for The Loomba Foundation – Kajol. C